Deel XVI Getaltheorie ingevulde versie

Page 1

Wiskunde In zicht een cursus wiskunde voor studierichtingen met component wiskunde derde graad algemeen secundair onderwijs geschreven door

Koen De Naeghel Deel XVI Getaltheorie (ingevulde versie)

15/08/2021


CREATIVE COMMONS Naamsvermelding-NietCommercieel-GelijkDelen 3.0 (CC BY-NC-SA) Dit is de vereenvoudigde (human-readable) versie van de volledige licentie. De volledige licentie is beschikbaar op de webpagina http://creativecommons.org/licenses/by-nc-sa/3.0/nl/legalcode De gebruiker mag: het werk kopiëren, verspreiden en doorgeven Remixen - afgeleide werken maken

Onder de volgende voorwaarden: Naamsvermelding - De gebruiker dient bij het werk de door de maker of de licentiegever aangegeven naam te vermelden (maar niet zodanig dat de indruk gewekt wordt dat zij daarmee instemmen met je werk of je gebruik van het werk). Niet-commercieel - De gebruiker mag het werk niet voor commerciële doeleinden gebruiken. Gelijk delen - Indien de gebruiker het werk bewerkt kan het daaruit ontstane werk uitsluitend krachtens dezelfde licentie als de onderhavige licentie of een gelijksoortige licentie worden verspreid.

Met inachtneming van: Afstandname van rechten - De gebruiker mag afstand doen van een of meerdere van deze voorwaarden met voorafgaande toestemming van de rechthebbende. Publiek domein - Indien het werk of een van de elementen in het werk zich in het publieke domein onder toepasselijke wetgeving bevinden, dan is die status op geen enkele wijze beı̈nvloed door de licentie. Overige rechten - Onder geen beding worden volgende rechten door de licentie-overeenkomst in het gedrang gebracht: • Het voorgaande laat de wettelijke beperkingen op de intellectuele eigendomsrechten onverlet. • De morele rechten van de auteur. • De rechten van anderen, ofwel op het werk zelf ofwel op de wijze waarop het werk wordt gebruikt, zoals het portretrecht of het recht op privacy. Let op - Bij hergebruik of verspreiding dient de gebruiker de licentievoorwaarden van dit werk kenbaar te maken aan . derden door middel van een link naar http://creativecommons.org/licenses/by-nc-sa/3.0/nl/

Eerste druk: 2015 Versie: 15 augustus 2021 Gepubliceerd door: Online publicatie platform Issuu.com Auteursrecht omslagfoto: stylephotographs/123RF Stockfoto http://nl.123rf.com/profile stylephotographs Tekstzetsysteem: LATEX Royalty percentage: 0% c Koen De Naeghel, gelicenseerd onder een Creative Commons Naamsvermelding-NietCommercieel-GelijkDelen 3.0


Deel XVI

Algebra - Getaltheorie

XVI


Inhoudsopgave

Deel Getaltheorie

1 Inleidende begrippen en definities

1

1.1 1.2

Verzamelingen . . . . . . . . . . . . . . . . . . . . . . . . . . . Eigenschappen van natuurlijke en gehele getallen . . . . . . . . Optelling van getallen - De groep Z, + . . . . . . . . . . . . . . Optelling van getallen - De ring Z, +, · . . . . . . . . . . . . . . Ordening van getallen - De geordende ring Z, +, ·, ≤ . . . . . . Principe van inductie . . . . . . . . . . . . . . . . . . . . . . . Principe van welordening - De welgeordende verzameling N, ≤ Oefeningen . . . . . . . . . . . . . . . . . . . . . . . . . . . . . . . .

. . . . . . . .

. . . . . . . .

. . . . . . . .

. . . . . . . .

. . . . . . . .

. . . . . . . .

. . . . . . . .

. . . . . . . .

. . . . . . . .

. . . . . . . .

. . . . . . . .

. . . . . . . .

. . . . . . . .

. . . . . . . .

. . . . . . . .

. . . . . . . .

. . . . . . . .

. . . . . . . .

. . . . . . . .

. . . . . . . .

. . . . . . . .

. . . . . . . .

. . . . . . . .

. . . . . . . .

. . . . . . . .

. . . . . . . .

. . . . . . . .

. . . . . . . .

. . . .

. . . .

. . . .

. . . .

. . . .

. . . .

. . . .

. . . .

. . . .

. . . .

. . . .

. . . .

. . . .

. . . .

. . . .

. . . .

. . . .

. . . .

. . . .

. . . .

. . . .

. . . .

. . . .

. . . .

. . . .

. . . .

. . . .

. 8 . 11 . 14 . 16

2 Deelbaarheid van gehele getallen 2.1 Deelbaarheid en delers . . . . . . . 2.2 Deling met rest . . . . . . . . . . . 2.3 Veelvouden en lineaire combinaties Oefeningen . . . . . . . . . . . . . . . .

1 3 3 3 4 5 5 6

8 . . . .

. . . .

. . . .

. . . .

. . . .

. . . .

. . . .

. . . .

. . . .

. . . .

. . . .

. . . .

. . . .

. . . .

. . . .

. . . .

3 Priemgetallen

19

3.1 Eenheden, priemgetallen en samengestelde getallen . . . . . . . . . . . . . . . . . . . . . . . . . . . . . . . . . . . 19 3.2 Hoofdstelling van de rekenkunde . . . . . . . . . . . . . . . . . . . . . . . . . . . . . . . . . . . . . . . . . . . . . 26 Oefeningen . . . . . . . . . . . . . . . . . . . . . . . . . . . . . . . . . . . . . . . . . . . . . . . . . . . . . . . . . . . . 30

4 Grootste gemene delers en kleinste gemene veelvouden 4.1 4.2 4.3

Grootste gemene delers . . . . . Kleinste gemene veelvouden . . Toepassingen . . . . . . . . . . . Toepassing 1 - Fermatgetallen . Toepassing 2 - Mersennegetallen Toepassing 3 - Tandwielen . . . Oefeningen . . . . . . . . . . . . . . .

. . . . . . .

. . . . . . .

. . . . . . .

. . . . . . .

. . . . . . .

. . . . . . .

. . . . . . .

. . . . . . .

. . . . . . .

. . . . . . .

. . . . . . .

. . . . . . .

. . . . . . .

. . . . . . .

. . . . . . .

. . . . . . .

. . . . . . .

32 . . . . . . .

. . . . . . .

. . . . . . .

. . . . . . .

. . . . . . .

. . . . . . .

. . . . . . .

. . . . . . .

. . . . . . .

. . . . . . .

. . . . . . .

. . . . . . .

. . . . . . .

. . . . . . .

. . . . . . .

. . . . . . .

. . . . . . .

. . . . . . .

. . . . . . .

. . . . . . .

. . . . . . .

. . . . . . .

. . . . . . .

. . . . . . .

. . . . . . .

. . . . . . .

. . . . . . .

. . . . . . .

32 38 40 40 43 44 45

Antwoorden op geselecteerde oefeningen

50

Referentielijst

53


Hoofdstuk 1

Inleidende begrippen en definities In dit eerste hoofdstuk definiëren we enkele belangrijke begrippen en leggen we de notatie en terminologie vast die in het vervolg van dit deel voortdurend zal gebruikt worden. De definities en eigenschappen zijn erg eenvoudig, zodat dit hoofdstuk grotendeels als zelfstudie aan de leerling kan worden overgelaten.1

1.1

Verzamelingen

In deze paragraaf overlopen we de belangrijkste kenmerken van een van de meest fundamentele begrippen in de wiskunde: verzamelingen. 3 Afspraak. Een verzameling kan omschreven worden als een collectie objecten, die we haar elementen noemen. Een verzameling heeft geen ordening en elk element kan hoogstens één keer voorkomen. We kunnen een verzameling beschrijven door haar elementen in een willekeurige volgorde op te sommen. Het geheel van alle elementen wordt tussen accolades geplaatst. We stellen een verzameling schematisch voor aan de hand van een Venndiagram.2 Voorbeeld. A = {−2, 1, 3} is een verzameling. Omdat een verzameling geen ordening heeft, zal bijvoorbeeld {3, −2, 1} dezelfde verzameling A zijn. De verzameling A kan als volgt worden voorgesteld:

A

−2 1 3 3 Notatie. Beschouw een verzameling A. Als een element a tot A behoort, dan schrijven we a ∈ A, wat we lezen als a is een element van A. In het andere geval schrijven we a ∈ / A. Bevat A een eindig aantal elementen, dan noteren we dat aantal met #A. 3 Definitie. De unieke verzameling zonder elementen noemen we de lege verzameling, genoteerd met {} of met ∅.

3 Definitie. De verzameling van de natuurlijke getallen is {0, 1, 2, 3, . . .} en wordt genoteerd met N. We maken de afspraak dat 0 een element is van N en we noteren de verzameling {1, 2, 3, . . .} als N0 . De verzameling van de gehele getallen is {0, 1, −1, 2, −2, . . .}, genoteerd met Z. In symbolen:

N = {0, 1, 2, 3, . . .}

N0 = {1, 2, 3, . . .} Z = {0, 1, −1, 2, −2, 3, −3, . . .} 3 Afspraak. Een andere mogelijkheid om een verzameling te beschrijven is het geven van een of meerdere eigenschappen waaraan de elementen van die verzameling moeten voldoen. De verticale streep | wordt gelezen als waarvoor geldt. Voorbeeld. De verzameling {a ∈ Z | a2 = 4} is dezelfde als {2, −2}.

3 Definitie. Zij A en B verzamelingen. Dan zeggen we dat A een deelverzameling is van B, genoteerd met A ⊆ B, indien elk element van A ook een element is van B. In symbolen: A⊆B

∀x ∈ A : x ∈ B

In dat geval zeggen we: de verzameling B omvat de verzameling A. Is dat niet zo, dan schrijven we A 6⊆ B.

1 Dit

hoofdstuk is gebaseerd op [5] (met toestemming van beide auteurs) en andere of eerdere uitgaven van Wiskunde In zicht. naar John Venn 1880. Gelijkaardige diagrammen werden eerder gebruikt door Leonhard Euler 18e eeuw, Gottfried Willhelm Leibniz 17e eeuw en Ramon Llull 13e eeuw. 2 Genoemd

XVI-1


Voorbeeld. Elk natuurlijk getal is een geheel getal, dus de verzameling van alle natuurlijke getallen is een deelverzameling van de verzameling van alle gehele getallen. Anders gezegd: De verzameling van de gehele getallen omvat de verzameling van de natuurlijke getallen. In symbolen: N ⊆ Z. Schematisch:

Z N 0 1 2 ...

−1 −2 ...

3 Opmerking. Beschouw een verzameling A. Omdat de lege verzameling geen enkel element bevat, is voldaan aan de uitspraak ∀x ∈ ∅ : x ∈ A. Dus ∅ ⊆ A, met andere woorden: elke verzameling omvat de lege verzameling. Hierboven hebben we al aangegeven dat {−2, 1, 3} en {3, −2, 1} dezelfde verzameling voorstellen. De volgende definitie geeft betekenis aan het begrip gelijkheid van verzamelingen. Daarnaast kunnen we met de bewerkingen doorsnede, unie en verschil nieuwe verzamelingen maken. 3 Definitie. Twee verzamelingen A en B zijn gelijk als A een deelverzameling is van B en als B een deelverzameling is van A. We schrijven dan A = B. Is dat niet zo, dan schrijven we A 6= B. 3 Definitie. Zij A en B verzamelingen. De doorsnede van A en B is de verzameling van alle elementen die in A en in B bevat zijn. We noteren die verzameling met A ∩ B. In symbolen:

A ∩ B = {x | x ∈ A en x ∈ B} Schematisch:

A

B A∩B

3 Definitie. De unie van A en B, genoteerd met A ∪ B, is de verzameling van alle elementen die in A of in B bevat zijn. In symbolen: A ∪ B = {x | x ∈ A of x ∈ B} Schematisch:

A

B A∪B

3 Definitie. Het verschil van A met B is de verzameling van alle elementen die wel tot A maar niet tot B behoren. Die verzameling wordt met A \ B genoteerd. In symbolen:

A\B = {x | x ∈ A en x ∈ / B} Schematisch:

A

B

A\B

XVI-2


1.2

Eigenschappen van natuurlijke en gehele getallen

In deze paragraaf overlopen we de meest elementaire eigenschappen van de gehele getallen. De lezer kan zelf nagaan welke van deze eigenschappen ook voor de natuurlijke getallen gelden.

Optelling van getallen - De groep Z, + 3 Eigenschap 1. De optelling van de gehele getallen voldoet aan de volgende eigenschappen: (1) (2)

optelling is inwendig: optelling is associatief:

(3)

het getal 0 is het neutraal element voor optelling:

(4)

invers element voor optelling:

∀a, b ∈ Z : a + b ∈ Z ∀a, b, c ∈ Z : (a + b) + c = a + (b + c) ∀a ∈ Z : a + 0 = a = 0 + a

∀a ∈ Z : ∃b ∈ Z : a + b = 0 = b + a.

Je kan eenvoudig aantonen dat elk geheel getal a precies één invers element voor de optelling heeft. Voortaan zullen we dat invers element van a noteren met −a.

Omdat voldaan is aan eigenschappen (1)-(4) noemen we de verzameling Z voorzien van de optelling een groep, notatie Z, +. Bovendien geldt ook de eigenschap (5)

∀a, b ∈ Z : a + b = b + a.

optelling is commutatief:

Wegens eigenschap (5) noemen we de groep Z, + commutatief (of abels).3 Omdat Z, + een groep is, geldt de zogenaamde (6)

∀a, b, c ∈ Z : a + b = a + c ⇒ b = c.

schrapwet voor de optelling:

Bewijs van (6). Neem a, b, c ∈ Z. We hebben: a+b=a+c

(−a) + (a + b) = (−a) + (a + c)

((−a) + a) + b = ((−a) + a) + c

b = c.

0+b=0+c

Bij afspraak is het verschil a − b van twee gehele getallen a en b gelijk aan de som a + (−b). Hieruit volgen de bekende tekenregels zoals a − (b + c) = a − b − c en a − (b − c) = a − b + c.

Vermenigvuldiging van getallen - De ring Z, +, · 3 Eigenschap 2. De vermenigvuldiging van de gehele getallen voldoet aan de volgende eigenschappen:4 ∀a, b ∈ Z : a · b ∈ Z

(7)

vermenigvuldiging is inwendig:

(8)

vermenigvuldiging is associatief:

(9)

vermenigvuldiging is distributief ten opzichte van optelling:

∀a, b, c ∈ Z : (a · b) · c = a · (b · c)

∀a, b, c ∈ Z : a · (b + c) = a · b + a · c ∀a, b, c ∈ Z : (a + b) · c = a · c + b · c

(10) vermenigvuldiging is commutatief: (11) het getal 1 is neutraal element voor vermenigvuldiging:

∀a, b ∈ Z : a · b = b · a

∀a ∈ Z : a · 1 = a = 1 · a.

Omdat de commutatieve groep Z, + voldoet aan eigenschappen (7)-(11) noemen we de verzameling Z voorzien van de optelling en de vermenigvuldiging een commutatieve ring met eenheidselement, notatie Z, +, ·. Bovendien geldt ook de eigenschap ∀a, b ∈ Z : ab = 0 ⇒ a = 0 of b = 0.

(12) er zijn geen nuldelers:

Wegens eigenschap (12) noemen we de ring Z, +, · een integriteitsdomein, of kortweg domein. Omdat de ring Z, +, · een domein is, geldt ook de ∀a, b, c ∈ Z : ab = ac en a 6= 0 ⇒ b = c.

(13) schrapwet voor de vermenigvuldiging: Bewijs van (13). Neem a, b, c ∈ Z. We hebben: ab = ac

⇒ ⇒

ab − ac = 0 a(b − c) = 0.

Vermits a 6= 0 moet dan b − c = 0 (waarom?) dus b = c. 3 Genoemd 4 De

naar Niels Henrik Abel (1802 - 1829). vermenigvuldiging van twee gehele getallen a en b wordt zowel met a · b als met ab genoteerd.

XVI-3


Ordening van getallen - De geordende ring Z, +, ·, ≤

3 Eigenschap 3. De relatie kleiner dan of gelijk aan van de gehele getallen voldoet aan de volgende eigenschappen: ∀a ∈ Z : a ≤ a

(14) kleiner dan of gelijk aan is reflexief:

∀a, b ∈ Z : a ≤ b en b ≤ a ⇒ a = b

(15) kleiner dan of gelijk aan is antisymmetrisch:

∀a, b, c ∈ Z : a ≤ b en b ≤ c ⇒ a ≤ c.

(16) kleiner dan of gelijk aan is transitief:

Omdat voldaan is aan eigenschappen (14)-(16), noemen we de verzameling Z voorzien van kleiner of gelijk aan een partieel geordende verzameling. Bovendien geldt ook de eigenschap ∀a, b ∈ Z : a ≤ b of b ≤ a.

(17) kleiner dan of gelijk aan is totaal:

Wegens eigenschap (17) noemen we de verzameling Z voorzien van kleiner dan of gelijk aan een totaal geordende verzameling. Het ordenen van getallen is verenigbaar met de optelling en vermenigvuldiging, en wel op de volgende manier. 3 Eigenschap 4. Voor elke a, b, c ∈ Z geldt: (18) als a ≤ b dan is a + c ≤ b + c

(19) als 0 ≤ a en 0 ≤ b dan is 0 ≤ ab. Daarom spreken we van de geordende ring Z, +, ·, ≤. De relaties (strikt) kleiner dan, groter dan of gelijk aan en (strikt) groter dan kennen de volgende beschrijving voor alle gehele getallen a en b: a<b a≥b

a>b

a ≤ b en a 6= b

a ≥ b en a 6= b

b≤a

De begrippen positief, negatief en hun negaties hebben de volgende betekenis voor alle gehele getallen a: a is positief a is negatief a is strikt positief a is strikt negatief

⇔ ⇔ ⇔

a≥0 a≤0 a>0 a<0

a is niet-negatief a is niet-positief

Voorbeeld. De verzameling van de strikt positieve gehele getallen gelijk is aan N0 , want {a ∈ Z | a > 0} = {1, 2, 3, . . .} = N0 . Voor een geheel getal a wordt de absolute waarde van van a, genoteerd met |a|, als volgt gedefinieerd: |a| =

ß

a −a

als a ≥ 0 als a < 0

Ten slotte vermelden we nog enkele andere eigenschappen van de geordende ring Z, +, ·, ≤. 3 Gevolgen. Zij a, b, c ∈ Z. Dan geldt: (20) als a ≤ b en 0 ≤ c dan is ac ≤ bc

(21) als a ≤ b en c ≤ 0 dan is ac ≥ bc (22) |ab| = |a| · |b|

(23) als a 6= 0 dan is |a| ≥ 1.

4 Een

reëel getal x wordt in België positief genoemd als x ≥ 0, terwijl in Nederland positief betekent: x > 0. Zie [4].

XVI-4


De partieel geordende verzameling N, ≤ voldoet aan twee belangrijke eigenschappen die we in het vervolg regelmatig zullen gebruiken. Formeel gezien zijn deze principes equivalent: de ene eigenschap kan met behulp van de andere eigenschap worden aangetoond en omgekeerd.

Principe van inductie 3 Eigenschap (principe van inductie). Zij V ⊆ N die voldoet aan de volgende twee voorwaarden: (1) 0 ∈ V , en (2) voor elke k ∈ N: als k ∈ V dan is k + 1 ∈ V .

Dan is V = N.

Deze eigenschap leidt tot een manier om eigenschappen van natuurlijke getallen te bewijzen. 3 Modelvoorbeeld. Bewijs dat voor alle strikt positieve natuurlijke getallen n geldt dat 1 + 2 + 4 + 8 + · · · + 2n−1 = 2n − 1. Bewijstechniek. Een eigenschap die geldt voor elk natuurlijk getal n ≥ n0 wordt vaak elegant aangetoond door middel van een zogenaamd bewijs met inductie op n. Zo’n bewijs bestaat uit twee delen. In het eerste deel, dat de inductiebasis wordt genoemd, wordt aangetoond dat de eigenschap geldt voor n = n0 . In het tweede deel, de inductiestap, wordt aangetoond dat indien de eigenschap geldt voor een bepaalde waarde van n (bijvoorbeeld n = k) hij ook geldt voor de volgende waarde van n (dit is dan n = k + 1). De veronderstelling dat de te bewijzen eigenschap geldt voor n = k noemen we de inductiehypothese. Als we dit hebben kunnen aantonen, dan is de stelling bewezen. Immers, we kunnen als volgt redeneren. De eigenschap is waar voor n0 (dat was de inductiebasis) dus volgt uit de inductiestap dat de eigenschap waar is voor n0 + 1. Maar dan volgt, opnieuw wegens de inductiestap, dat de eigenschap waar is voor n0 + 2, enzovoort. Daaruit volgt dat de eigenschap waar is voor alle n ≥ n0 . Bewijs. We bewijzen de formule voor alle n ≥ 1 met inductie op n. (i) Inductiebasis. Voor n = 1 is enerzijds 1 + 2 + 4 + 8 + · · · + 2n−1 = 1, terwijl anderzijds 2n − 1 = 2 − 1 = 1. (ii) Inductiestap. Stel dat de formule waar is voor n = k (met k ≥ 1), dus dat 1 + 2 + 4 + 8 + · · · + 2k−1 = 2k − 1. We moeten aantonen dat de formule ook waar is voor n = k+1, dus dat 1+2+4+8+· · ·+2k+1−1 = 2k+1 −1. Welnu, 1 + 2 + 4 + 8 + · · · + 2k+1−1 = 1 + 2 + 4 + 8 + · · · + 2k

= 1 + 2 + 4 + 8 + · · · + 2k−1 + 2k = 2k − 1 + 2k = 2 · 2k − 1

= 2k+1 − 1.

Uit (i) en (ii) volgt nu dat de formule geldt voor alle n ≥ 1.

Principe van welordening - De welgeordende verzameling N, ≤

3 Eigenschap (principe van welordening). Elke niet-lege deelverzameling V ⊆ N bevat een kleinste element.

Het principe van welordening is equivalent met het principe van inductie, dus elk probleem dat kan worden opgelost met inductie kan ook worden opgelost met welordening, en omgekeerd. Bij wijze van illustratie bewijzen we een andere elementaire eigenschap van de verzameling van de natuurlijke getallen met behulp van het principe van welordening. 3 Eigenschap (Archimedes). Als a, b ∈ N0 dan bestaat er een positief natuurlijk getal n waarvoor na ≥ b. Bewijs. Stel, uit het ongerijmde, dat er geen enkel positief natuurlijk getal n bestaat waarvoor na ≥ b. Met andere woorden, stel dat na < b voor elk positief natuurlijk getal n. Beschouw nu de verzameling V = {b − na | n ∈ N}.

Uit onze veronderstelling volgt dat V ⊆ N en merk op dat V niet leeg is (waarom?). Wegens het principe van de welordening bevat V een kleinste element x. Omdat x ∈ V is x = b − ma voor een zekere m ∈ N. Maar dan is b − (m + 1)a = b − ma − a < b − ma een element van V dat kleiner is dan x, een strijdigheid. Dus onze veronderstelling dat er geen enkel positief natuurlijk getal n bestaat waarvoor na ≥ b fout is. Met andere woorden, er is een positief natuurlijk getal n waarvoor na ≥ b. XVI-5


Oefeningen 1 Inleidende begrippen en definities

Basis ?

1.1 Verzamelingen

1 2

3

1.2 Eigenschappen van natuurlijke en gehele getallen

6 7

8

Verdieping ? ??

??

Uitbreiding ? ??

4 5 8

8

9

10

Oefeningen bij §1.1 B

B

B?

Oefening 1. Stel A = {1, 3, 5, 6} en B = {2, 3, 4, 6}. Beschrijf telkens de verzameling door middel van opsomming. (a) A ∩ B

(d) B \ A

(b) A ∪ B

(e) A \ A

(c) A \ B

(f) A \ (A ∩ B)

Oefening 2. Waar of vals? Beoordeel de volgende uitspraken. Verklaar telkens je antwoord. (a) ∅ = {0}

(e) ∅ ∈ {∅}

(b) 6 ∈ {6}

(f) {1, 3, 6, 7, 8} ⊆ {1, 3, 6, 7, 9}

(c) {2, 4, 6, 8, 10} ⊆ N

(g) ∅ ⊆ {3, 5, 7}

(d) ∅ = {∅}

(h) {0} ∈ Z

Oefening 3. Beschrijf telkens de gegeven verzameling door opsomming. (a) {y ∈ Z | 2y 2 = 50} (b) {x ∈ N | ∃m ∈ Z : x = 2m − 5} (c) {a ∈ Z | 3a2 = −12}

V

V

Oefening 4. Zij P en Q twee deelverzamelingen van Z waarvoor P ∪ Q = P . Wat kan je uit elk van de onderstaande gegevens besluiten over P en Q? (a) P ∩ Q = ∅

(c) P ∩ (Z \ Q) = ∅

(b) P ∪ Q = ∅

(d) (Z \ P ) ∪ Q = ∅

Oefening 5 (Vlaamse Wiskunde Olympiade 1987 eerste ronde). Als V = {a, b, {c, d}} dan geldt (A) c ∈ V (B) {c, d} ⊆ V (C) {a, b, c, d} ⊆ V (D) {{c, d}} ⊆ V (E) {c} ∈ V

XVI-6


Oefeningen bij §1.2 B

Oefening 6. Beschouw de verzameling V = {a ∈ Z | −5 < a ≤ 3}. (a) Beschijf de verzameling V door opsomming. (b) Bevat V een eindig aantal elementen? Zo ja, geef #A.

B

Oefening 7. Waar of vals? Beoordeel de volgende uitspraken. Indien waar, bewijs. Indien vals, geef een tegenvoorbeeld. (a) De verzameling N voorzien van de optelling is een groep. (b) De verzameling Z voorzien van de aftrekking is een groep. (c) Elk geheel getal heeft een invers element voor de vermenigvuldiging. (d) Elk niet-positief geheel getal is negatief. (e) Elk positief geheel getal is niet-negatief. (f) Als a, b ∈ Z dan is |a + b| = |a| + |b|. (g) De verzameling Z voorzien van de relatie kleiner dan of gelijk aan voldoet aan het principe van welordening. Oefening 8. Bewijs telkens met inductie de formule voor alle strikt positieve natuurlijke getallen n. B? (a) 1 + 2 + 3 + 4 + · · · + n =

1 n(n + 1) 2

B?? (c) 1 + 3 + 9 + 27 + · · · + 3n−1 =

V (d) (1 + 2 + 3 + · · · + n)2 = 13 + 23 + 33 + · · · + n3

B? (b) 1 + 3 + 5 + 7 + · · · + (2n − 1) = n2 V?

1 n (3 − 1) 2

Oefening 9 (Nederlandse Wiskunde Olympiade 1986 finale). Bewijs dat voor alle strikt positieve gehele getallen n geldt: 1 1 1 1 1 1 + + ··· + = + + ··· + . 1·2 3·4 (2n − 1) · (2n) n+1 n+2 2n

U??

Oefening 10 (veeltermidentiteiten). Een identiteit is een uitdrukking van de vorm = 4, die waar is voor alle waarden van a, b, . . . of x, y, . . . die in of 4 voorkomen en waarvoor de opgave zin heeft. Is zowel het linkerlid als het rechterlid een veelterm, dan noemen we die identiteit een veeltermidentiteit. Zo zijn de volgende merkwaardige producten alle veeltermidentiteiten in twee variabelen a en b: (a + b)2 = a2 + 2ab + b2 (a − b)2 = a2 − 2ab + b2

(a + b)3 = a3 + 3ab2 + 3a2 b + b3 (a − b)3 = a3 − 3ab2 + 3a2 b − b3 a2 − b2 = (a − b)(a + b)

a3 − b3 = (a − b)(a2 + ab + b2 )

a3 + b3 = (a + b)(a2 − ab + b2 )

Vervangen we in de laatste drie identiteiten de variabele b door 1 en noemen we a nu x, dan verkrijgen we veeltermidentiteiten in één variabele x: x2 − 1 = (x − 1)(x + 1)

x3 − 1 = (x − 1)(x2 + x + 1) x3 + 1 = (x + 1)(x2 − x + 1)

Bewijs telkens met inductie de veeltermidentiteit. (a) xn − 1 = (x − 1)(xn−1 + xn−2 + · · · + x2 + x + 1) voor alle n ∈ N0 (b) x2n+1 + 1 = (x + 1)(x2n − x2n−1 + x2n−2 − x2n−3 + · · · + x2 − x + 1) voor alle n ∈ N XVI-7


Hoofdstuk 2

Deelbaarheid van gehele getallen In de getaltheorie staat de deelbaarheid van gehele getallen centraal. Na het invoeren van dit begrip zien we de meest belangrijke basiseigenschappen. Daarna komt het fundamenteel resultaat deling met rest aan bod. Ten slotte voeren we de concepten veelvoud en lineaire combinatie in.

2.1

Deelbaarheid en delers

In deze paragraaf bespreken we de definitie en enkele basiseigenschappen van deelbaarheid van gehele getallen. 3 Definitie. Zij a, b ∈ Z met b 6= 0. We noemen a (geheel) deelbaar door b als er een q ∈ Z bestaat waarvoor a = qb. In dat geval noemen we b een (gehele) deler van a, hetgeen we noteren als b | a. In symbolen:

b|a

∃q ∈ Z : qb = a

Is b geen deler van a, dan schrijven we b 6 | a. Deelbaarheid door 0 blijft ongedefinieerd, zodat de schrijfwijzen 0 | a en 06 | a geen betekenis hebben en dus zinloze uitspraken zijn. Voorbeeld. We hebben dat 3 | 15, 8 | −8

en

66 | 21.

Bij de volgende eigenschap bewijzen we enkel (iii), (v) en (viii). De bewijzen van basiseigenschappen (i), (ii), (iv), (vi) en (vii) worden als Oefening 9 voor de lezer gehouden. 3 Eigenschap (basiseigenschappen van deelbaarheid). Zij a, b, c, k, l ∈ Z en d, e ∈ Z0 . Dan geldt: (i) 1 | a en −1 | a

(ii) a 6= 0 ⇒ a | 0, a | a en −a | a

(iii) d | a ⇒ −d | a

(iv) d | a ⇒ d | ka en ed | ea

(v) d | a en d | b ⇒ d | (ka + lb)

(vi) d | a en d | b ⇒ d | ab

(vii) d | e en e | a ⇒ d | a

(viii) d | a ⇒ |d| ≤ |a| of a = 0. Bewijs van (iii). Veronderstel dat d | a. Dus a = qd voor een zekere q ∈ Z. We moeten aantonen dat −d | a, dus aan te tonen: a = q 0 (−d) voor een zekere q 0 ∈ Z. Welnu,

a = qd = (−q)(−d). Noem nu q 0 = −q. Dan is inderdaad a = q 0 (−d) voor een zekere q 0 ∈ Z.

XVI-8


Bewijs van (v). Veronderstel dat d | a en d | b. Dus a = qd voor een zekere q ∈ Z en b = q 0 d voor een zekere q 0 ∈ Z. We moeten aantonen dat d | (ka + lb), dus aan te tonen: ka + lb = q 00 d voor een zekere q 00 ∈ Z. Welnu,

ka + lb = k(qd) + l(q 0 d) = (kq + lq 0 )d. Noem nu q 00 = kq + lq 0 . Dan is inderdaad ka + lb = q 00 d voor een zekere q 00 ∈ Z.

Bewijs van (viii). Veronderstel dat d | a. Dus a = qd voor een zekere q ∈ Z. Aan te tonen: |d| ≤ |a| of a = 0.

Welnu,

a = qd

|a| = |qd|

|a| = |q| · |d|

Nu is q ∈ Z dus q = 0 of |q| ≥ 1, zie eigenschap (23) uit Hoofdstuk 1: . als q = 0 dan is: |a| = |0| · |d| =0 zodat a = 0; . als |q| ≥ 1 dan is: |a| = |q| · |d| ≥ 1 · |d| = |d| zodat |d| ≤ |a|. We besluiten dat |d| ≤ |a| of a = 0. Is a ∈ Z0 , dan volgt uit basiseigenschap (viii) van deelbaarheid dat elke deler d van a voldoet aan |d| ≤ |a|. Geldt voor a ∈ Z dat d | a en |d| < |a| dan noemen we d een echte deler van a. Voor het vervolg is het handig om een notatie voor de verzameling van alle delers van een geheel getal te voorzien. 3 Notatie. Zij a ∈ Z. De verzameling van alle delers van a wordt genoteerd met del(a). In symbolen:

del(a) = {b | b ∈ Z0 en b | a} Eens men de begrippen deelbaarheid en delers beheerst, dan kunnen typische gevolgen hiervan op eenvoudige manier bewezen worden. 3 Modelvoorbeeld. (a) Geef del(6), del(1), del(−12) en del(17). (b) Welk verband merk je op tussen de verzamelingen del(6) en del(−12)? Oplossing. (a) We hebben: del(6) = {1, −1, 2, −2, 3, −3, 6, −6}, del(1) = {1, −1}, del(−12) = {1, −1, 2, −2, 3, −3, 4, −4, 6, −6, 12, −12}, del(17) = {1, −1, 17, −17}. (b) Omdat 6 | 12 is elke deler van 6 ook een deler van 12, zodat del(6) ⊆ del(12). XVI-9


Een beschrijving van het aantal delers van een geheel getal volgt uit de basiseigenschappen van deelbaarheid. 3 Gevolg. Zij a ∈ Z. Dan geldt: (i) del(a) 6= ∅,

(ii) als a = 0 dan is del(a) = Z0 , (iii) als a 6= 0 dan bevat del(a) een eindig en even aantal elementen. Bewijs van (i). We hebben: 1|a

wegens basiseigenschap (i) van deelbaarheid

⇒ 1 ∈ del(a) ⇒

del(a) 6= ∅

Bewijs van (ii). Aan te tonen del(0) = Z0 , dus aan te tonen: del(0) ⊆ Z0 en Z0 ⊆ del(0). Alvast is del(0) ⊆ Z0 (leg uit). We moeten aantonen dat Z0 ⊆ del(0), dus aan te tonen: ∀b ∈ Z0 : b ∈ del(0). Neem b ∈ Z0 willekeurig. Welnu,

0=0·b

0 = qb

b|0

b ∈ del(0).

voor een zekere q ∈ Z

Bewijs van (iii). Stel dat a ∈ Z0 . Als d ∈ del(a), dan is |d| ≤ |a|

wegens basiseigenschap (viii) van deelbaarheid

parate kennis: voor elke x, c ∈ R is |x| ≤ c ⇔ −c ≤ x ≤ c ⇒

− |a| ≤ d ≤ |a|

d ∈ {1, −1, 2, −2, 3, −3, . . . , a, −a}.

Hieruit volgt dat del(a) ⊆ {1, −1, 2, −2, . . . , a, −a} zodat del(a) een deelverzameling van een eindige verzameling is. Dus del(a) is zelf ook een eindige verzameling. Verder geldt voor elke b ∈ Z: b ∈ del(a)

b|a

−b | a

−b ∈ del(a).

wegens basiseigenschap (iii) van deelbaarheid

Dit toont aan dat het aantal elementen van del(a) even is.

XVI-10


2.2

Deling met rest

Is een geheel getal niet deelbaar door een ander geheel getal verschillend van nul, dan moeten we ons tevreden stellen met een zogenaamde deling met rest, ook wel de euclidische deling genoemd. Omdat dit een fundamenteel resultaat is, plaatsen we de formulering ervan in een stelling. Het bewijs levert een algoritme om deling met rest uit te voeren.1 3 Stelling (deling met rest, euclidische deling). Zij a, b ∈ Z met b 6= 0. Dan bestaat er precies één geheel getal q en precies één geheel getal r zodat a = qb + r

0 ≤ r < |b| .

en

Het getal a wordt het deeltal genoemd, b de deler, q het quotiënt en r de rest van de deling van a door b. Bewijs. Beschouw de verzameling S = {a, a − b, a + b, a − 2b, a + 2b, a − 3b, a + 3b, . . .} = {a − mb | m ∈ Z} ⊆ Z. We beweren dat S natuurlijke getallen bevat. Inderdaad, ofwel is b > 0 ofwel is b < 0: . als b > 0 dan bekijken we de strikt stijgende rij a, a + b, a + 2b, a + 3b, . . . waarvan alle termen tot S behoren, en omdat b > 0 bevat deze rij getallen die tot N behoren; . als b < 0 dan bekijken we de strikt stijgende rij a, a − b, a − 2b, a − 3b, . . . waarvan alle termen tot S behoren, en omdat b < 0 bevat deze rij getallen die tot N behoren. Wegens het principe van welordening bevat S een kleinste natuurlijk getal r. Omdat r ∈ S geldt dat voor een zekere q ∈ Z.

r = a − qb

Dus er bestaat een geheel getal q en een geheel getal r zodat a = qb + r

en

0 ≤ r.

We beweren dat r < |b|. Inderdaad, mocht r ≥ |b|, noem dan r0 = r − |b|. Dan is: (1) r0 ∈ N want r ≥ |b| dus r0 = r − |b| ≥ 0; (2) r0 ∈ S want ofwel is b > 0 ofwel is b < 0: . als b > 0 dan is: r0 = r − |b| =r−b = a − qb − b = a − (q + 1)b ∈ S; . als b < 0 dan is: r0 = r − |b| = r − (−b) = a − qb + b = a − (q − 1)b ∈ S; 1 Een

algoritme kan worden omschreven als een eindige reeks instructies om vanuit een gegeven begintoestand een probleem op te lossen of een taak uit te voeren. De fundamentele vraag welke problemen met een algoritme kunnen opgelost worden, werd in 1936 opgelost door Alan Turing , zie [12]. Zijn werk lag aan de oorsprong van een van de grootste uitvindingen ooit: de computer.

XVI-11


(3) r0 < r want r0 = r − |b| en b 6= 0.

Maar dan is r0 een natuurlijk getal in S dat kleiner is dan r. Een strijdigheid, want r het kleinste natuurlijk getal dat tot S behoort. We mogen dus besluiten dat r < |b|.

Dus er bestaat een geheel getal q en een geheel getal r zodat a = qb + r

en

0 ≤ r < |b| .

Om aan te tonen dat q en r uniek zijn met deze eigenschap, nemen we aan dat er ook nog een geheel getal q 00 en een geheel getal r00 bestaan zodat a = q 00 b + r00

en

0 ≤ r00 < |b| .

We moeten aantonen dat q 00 = q en r00 = r. Welnu, a=a

qb + r = q 00 b + r00

r − r00 = (q 00 − q)b

|r − r00 | = |q 00 − q| · |b| .

Nu is q 00 − q ∈ Z zodat q 00 − q = 0 of |q 00 − q| ≥ 1, zie eigenschap (23) uit Hoofdstuk 1: . als q 00 − q = 0 dan is: |r − r00 | = |0| · |b| =0 zodat r − r00 = 0; . als |q 00 − q| ≥ 1 dan is: |r − r00 | = |q 00 − q| · |b| ≥ 1 · |b| = |b| zodat |r − r00 | ≥ |b|. Maar anderzijds is ook |r − r00 | < |b|. Inderdaad, r < |b| r00 < |b|

en en

− r00 ≤ 0

r − r00 < |b|

−r ≤0

r00 − r < |b|

zodat Het geval |q 00 − q| ≥ 1 is dus onmogelijk.

|r − r00 | = ±(r − r00 ) < |b| .

We besluiten dat q 00 − q = 0 en r − r00 = 0, i.e. q 00 = q en r00 = r.

XVI-12


Merk op dat a ∈ Z deelbaar is door b ∈ Z0 als en slechts als de rest bij deling van a door b gelijk is aan nul. In dat geval spreken we van een opgaande deling. In het algemeen geeft het bewijs van de vorige stelling een manier om deling met rest van twee gehele getallen uit te voeren. 3 Modelvoorbeeld. Bepaal het quotiënt en de rest van de deling van −45 door 14.

Oplossing. Noem a = −45 en b = 14. We bepalen het kleinste natuurlijk getal r in de verzameling S = {a, a − b, a + b, a − 2b, a + 2b, a − 3b, a + 3b, . . .} = {−45, −45 − 14, −45 + 14, −45 − 2 · 14, −45 + 2 · 14, −45 − 3 · 14, −45 + 3 · 14, . . .} = {−45, −59, −31, −73, −17, −87, −3, −101, 11, −115, 25, . . .}. We vinden dat r = 11. Nu is r = a − qb

11 = −45 − q · 14

waarbij q = −4.

Hieruit volgt dat a = qb + r ⇒

en

0 ≤ r < |b|

− 45 = (−4) · 14 + 11

en

0 ≤ 11 ≤ |14| .

Dus de deling van −45 door 14 heeft als quotiënt q = −4 en als rest r = 11.

Alternatieve oplossing. We schrijven −45 als de som van een geheel aantal keer 14 met een restgetal, en manipuleren het veelvoud van 14 zodat het restgetal positief en kleiner dan 14 wordt. −45 = 0 · 14 + (−45) = (−1) · 14 + (−31) = (−2) · 14 + (−17) = (−3) · 14 + (−3) = (−4) · 14 + 11

en

0 ≤ 11 ≤ |14| .

Dus de deling van −45 door 14 heeft als quotiënt q = −4 en als rest r = 11.

Zijn deeltal en deler strikt positieve gehele getallen, dan kan het quotiënt en de rest bij deling van a door b sneller gevonden worden met de klassieke staartdeling uit het lager onderwijs. Bij de aanvankelijke opzet ging het daarbij om de deling van een (meestal groot) natuurlijk getal door een kleiner natuurlijk getal. Het principe kan echter ook voor andere delingen gebruikt worden, bijvoorbeeld voor de deling van veeltermen. 3 Modelvoorbeeld 2. Bepaal het quotiënt en de rest bij deling van 2405 door 11 door middel van staartdeling. Oplossing.

XVI-13


2.3

Veelvouden en lineaire combinaties

De begrippen veelvoud en deler houden nauw met elkaar verband. 3 Definitie. Zij a, b ∈ Z. We noemen a een (geheel) veelvoud van b als er een q ∈ Z bestaat waarvoor a = bq. De gehele veelvouden van 2 zijn precies de even getallen, de overige gehele getallen zijn oneven. Voor het vervolg is het handig om een notatie voor de veelvouden van een willekeurig geheel getal a te voorzien. 3 Notatie. Zij a ∈ Z. De verzameling van alle veelvouden van a noteren we met aZ. In symbolen:

aZ = {aq | q ∈ Z} Voorbeeld. We hebben (vul aan): 5Z = {0, 5, −5, 10, −10, 15, −15, 20, −20, . . .}, (−5)Z = {0, −5, 5, −10, 10, −15, 15, −20, 20, . . .} = 5Z, 1Z = Z, 0Z = {0}. De verzameling aZ van alle gehele veelvouden van een geheel getal a kan als volgt worden veralgemeend. 3 Definitie. Zij a, b ∈ Z. Een (gehele) lineaire combinatie van a en b is een geheel getal van de gedaante am + bn

waarbij

m, n ∈ Z.

De verzameling van alle lineaire combinaties van a en b noteren we met aZ + bZ. In symbolen: aZ + bZ = {am + bn | m, n ∈ Z} Voorbeeld. We hebben (vul aan): (−20)Z + 12Z = {(−20)m + 12n | m, n ∈ Z} = {−20, 12, −20 + 12, −20 − 12, 20 + 12, 20 − 12, −20 + 24, −20 − 24, 20 + 24, 20 − 24, . . .} = {−20, 12, −8, −32, 32, 8, 4, −48, 48, −4, −28, −52, 52, 28, . . .}

We beweren dat (−20)Z + 12Z = 4Z. Om dat te bewijzen, zullen we aantonen dat enerzijds 4Z ⊆ (−20)Z + 12Z en dat anderzijds (−20)Z + 12Z ⊆ 4Z. . Is 4Z ⊆ (−20)Z + 12Z? Neem x ∈ 4Z willekeurig. Dan is x = 4q voor een zekere q ∈ Z. Welnu, x = 4q = (−20 · 1 + 12 · 2)q = (−20)q + 12 · (2q) ∈ (−20)Z + 12Z. . Is (−20)Z + 12Z ⊆ 4Z? Neem y ∈ (−20)Z + 12Z willekeurig. Dan is y = (−20)m + 12n voor zekere m, n ∈ Z. Welnu, y = (−20)m + 12n = 4(−5m + 3n) ∈ 4Z.

XVI-14


Hieronder zullen we dit voorbeeld veralgemenen tot een hulpstelling, ook wel een lemma genoemd. Dat doen we omdat we deze hulpstelling laten zullen gebruiken om een andere, belangrijke stelling te bewijzen. 3 Lemma 1. Zij a, b ∈ Z. Dan is er een geheel getal d waarvoor aZ + bZ = dZ. Bewijs. Als a = 0 en b = 0 dan volstaat de keuze d = 0. Inderdaad, als a = 0 en b = 0 dan is aZ + bZ = {am + bn | m, n ∈ Z} = {0 · m + 0 · n | m, n ∈ Z} = {0 | m, n ∈ Z} = {0} = 0Z Dus voor het vervolg van het bewijs mogen we aannemen dat a en b niet beide nul zijn. Nu bevat de verzameling aZ + bZ minstens één strikt positief geheel getal. Inderdaad, zo is bijvoorbeeld a2 + b2 ∈ aZ + bZ en a2 + b2 > 0 want a en b zijn niet beide nul. Wegens het principe van welordening bevat aZ + bZ een kleinste strikt positief geheel getal. Noem dat getal d. Omdat d ∈ aZ + bZ is d = ak + bl voor zekere k, l ∈ Z.

We beweren dat aZ + bZ = dZ. Om dat te bewijzen, zullen we aantonen dat enerzijds dZ ⊆ aZ + bZ en dat anderzijds aZ + bZ ⊆ dZ. . Is dZ ⊆ aZ + bZ? Neem x ∈ dZ willekeurig. Dan is x = dq voor een zekere q ∈ Z. Welnu, x = dq = (ak + bl)q = a(kq) + b(lq) ∈ aZ + bZ. . Is aZ + bZ ⊆ dZ? Neem y ∈ aZ + bZ willekeurig. Dan is y = am + bn voor zekere m, n ∈ Z. Wegens de stelling van de euclidische deling bestaat er precies één geheel getal q 0 en één geheel getal r zodat y = q0 d + r

en

0 ≤ r < |d| .

Merk op dat |d| = d want d > 0. We beweren dat r = 0. Inderdaad, mocht r 6= 0 dan zou

r = y − q0 d = am + bn − q 0 (ak + bl) = a(m − q 0 k) + b(n − q 0 l) ∈ aZ + bZ. Maar dan is r een strikt positief geheel getal in aZ + bZ dat kleiner is dan d. Een strijdigheid, want d is het kleinste strikt positief geheel getal in aZ + bZ. We mogen dus besluiten dat r = 0. Welnu, y = q 0 d + 0 = q 0 d ∈ dZ.

Analoog kan het volgend resultaat aangetoond worden. Het bewijs laten we als Oefening 36 voor de lezer. 3 Lemma 2. Zij a, b ∈ Z. Dan is er een geheel getal v waarvoor aZ ∩ bZ = vZ. XVI-15


Oefeningen 2 Deelbaarheid van gehele getallen

Basis ?

Verdieping ? ??

??

2.1 Deelbaarheid en delers

1 2

3 4

5

6 7

8

2.2 Deling met rest

10 11 12 13

14 15

16 17

18 19 20

21 22

2.3 Veelvouden en lineaire combinaties

23 24 25

26 27

28

29

30

Uitbreiding ? ?? 9

31

32 33 34

35

36 37

Oefeningen bij §2.1 B

Oefening 1. Bepaal alle gehele getallen n die een deler zijn van 5n − 12.

B

Oefening 2. Waar of vals? Beoordeel de volgende uitspraken. Verklaar telkens je antwoord. (a) Elk geheel getal heeft minstens vier delers. (b) Elk geheel getal heeft minstens één echte deler. (c) Elk geheel getal heeft een eindig aantal delers. (d) Voor elk geheel getal a geldt: del(a) = {b | b ∈ Z en b | a}. (e) Voor elk geheel getal a geldt: a ∈ del(a).

B?

Oefening 3. Bewijs de volgende uitspraken. (a) Zij a, b ∈ Z met b 6= 0. Dan geldt: b | a ⇔ ∃! q ∈ Z : qb = a. (b) Zij d, d0 ∈ Z0 en stel dat d | d0 en d0 | d. Dan is d = d0 of d = −d0 .

B?

Oefening 4 (Vlaamse Wiskunde Olympiade 1991 eerste ronde). Het geheel getal q waarvoor 5050 = 2525 q is gelijk aan (A) 2525

B??

(B) 1025

(C) 10025

(D) 225

(E) 2 · 2525

Oefening 5. (a) Stel dat a ∈ Z en dat del(a) = del(10). Waaraan kan a dan gelijk zijn? (b) Formuleer en bewijs een veralgemening van (a) voor willekeurige gehele getallen.

V

Oefening 6. Toon aan dat 20153 − 19773 deelbaar is door 19.

V

Oefening 7. Zij n een oneven natuurlijk getal. Bewijs dat 8 | n2 − 1.

V?

Oefening 8. Zij a ∈ Z en n ∈ N0 . Bewijs dat a − 1 een deler is van an − 1.

U

Oefening 9 (basiseigenschappen van deelbaarheid). Zij a, b, c, k, l ∈ Z en d, e ∈ Z0 . Bewijs volgende uitspraken. (i) 1 | a en −1 | a (ii) a 6= 0 ⇒ a | 0, a | a en −a | a (iv) d | a ⇒ d | ka en ed | ea (vi) d | a en d | b ⇒ d | ab (vii) d | e en e | a ⇒ d | a XVI-16


Oefeningen bij §2.2 B

Oefening 10. Bepaal telkens het quotiënt en de rest bij deling van a door b. (a) a = 60 en b = 7

(d) a = 27 en b = −8

(b) a = −60 en b = 7

(e) a = −64 en b = −8

(c) a = −34 en b = 5

(f) a = 15 en b = −28

B

Oefening 11. Bepaal alle gehele getallen die het deeltal zijn van een deling door 5, die 18 als quotiënt opleveren.

B

Oefening 12. Het quotiënt van een opgaande deling van twee gehele getallen is −24. Als men het deeltal verdubbelt en 5 bij de deler optelt, dan gaat de deling nog steeds op en is het quotiënt van deling gelijk aan 60. Welke zijn de deler en het deeltal?

B

Oefening 13. De deling van een geheel getal door 73 heeft als rest 3. Deelt men hetzelfde geheel getal door 70, dan is de rest 48. De quotiënten van beide delingen zijn gelijk. Bepaal dat geheel getal.

B?

Oefening 14. Maak de deling met rest van 284 door 23, en bepaal het grootste geheel getal dat men bij het deeltal mag optellen of aftrekken zonder dat het quotiënt bij deling door 23 verandert.

B?

Oefening 15. Van een deling met rest is de deler gelijk aan −2136 en het quotiënt gelijk aan 49. (a) Bepaal het deeltal waarvoor de rest bij deling van deler door deeltal zo groot mogelijk is. (b) Bepaal het deeltal waarvoor de rest bij deling van deler door deeltal zo klein mogelijk is.

B??

Oefening 16. Als twee gehele getallen bij deling door hun verschil dezelfde resten overlaten, dan verschillen de quotiënten met 1. Bewijs.

B??

Oefening 17. Een deling laat een rest 7 over. Telt men 1 op bij het deeltal, dan wordt het quotiënt 1 groter. Bepaal alle mogelijkeden voor de deler.

V

Oefening 18. Men deelt een geheel getal door 4, de rest is gelijk aan 2. Als men dat getal door 6 deelt, dan is de rest nooit 3. Bewijs.

V

Oefening 19. Een deling door 58 laat 40 tot rest over. Tussen welke getallen ligt het natuurlijk getal, dat men van het deeltal moet aftrekken, om het quotiënt met 1 te verkleinen?

V

Oefening 20 (Vlaamse Wiskunde Olympiade 1987 eerste ronde). a, n ∈ N0 en an geeft bij deling door 73 rest 2, an+1 geeft bij deling door 73 rest 69. Voor de rest r bij deling van a door 73 geldt (A) 0 ≤ r < 10

(B) 10 ≤ r < 30

(C) 30 ≤ r < 50

(D) 50 ≤ r < 70

(E) r ≥ 70

V?

Oefening 21. Bepaal twee gehele getallen waarvan de som 266 is en hun quotiënt bij deling met rest gelijk is aan 13. Geef nadien alle mogelijkheden voor die gehele getallen.

V?

Oefening 22. Zij a en b twee gehele getallen, noem q en r het quotiënt en r de rest bij deling van a door b en stel dat q 6= 0. Bewijs dat het quotiënt bij deling van a door q gelijk is aan de som van b en het quotiënt bij deling van r door q.

XVI-17


Oefeningen bij §2.3 B

Oefening 23. Waar of vals? Beoordeel de volgende uitspraken. Verklaar telkens je antwoord. (a) Er bestaat een geheel getal a waarvoor de verzameling aZ een eindig aantal elementen heeft. (b) Zij a, b ∈ Z waarvoor aZ = bZ. Dan is a = b.

B

Oefening 24. Bewijs de volgende uitspraken. (a) Zij a ∈ Z waarvoor 1 ∈ aZ. Dan is a = 1 of a = −1.

(b) Zij a ∈ Z0 . Dan geldt: aZ = {b ∈ Z | a is een deler van b}. B

Oefening 25. Bewijs dat het verschil van twee opeenvolgende kwadraten altijd een oneven getal is.

B?

Oefening 26 (Vlaamse Wiskunde Olympiade 1987 eerste ronde). Welke van de volgende vijf uitspraken is waar? (A) Het kwadraat van een oneven getal is soms even. (B) Als x even is, zijn x en 2x twee opeenvolgende even getallen. (C) Als x even is, is (x − 1)(x + 1) oneven. (D) Als x even is, is 107x soms oneven. (E) Als x en y oneven zijn, is 3(x + y) oneven.

B?

Oefening 27. Bewijs dat het verschil van twee opeenvolgende derdemachten altijd een oneven getal is.

B??

Oefening 28. Zij a, b ∈ Z. Toon aan:

a = ±b

aZ = bZ.

V

Oefening 29. Zij a een oneven geheel getal. Toon aan dat a2 − 1 ∈ 8Z.

V?

Oefening 30. Men deelt 18 door een veelvoud van 6, dat geen veelvoud van 18 is. Welke kan de rest zijn? Bewijs je antwoord.

V??

Oefening 31. Zij n ∈ N. Bewijs dat 32n+1 + 2n+2 een veelvoud is van 7.

U

Oefening 32 (kenmerk van deelbaarheid door 2 en door 5). Bewijs dat een natuurlijk getal deelbaar is door 2 als en slechts als het cijfer van de eenheden deelbaar is door 2, en deelbaar is door 5 als en slechts als het cijfer van de eenheden deelbaar is door 5.

U

Oefening 33 (kenmerk van deelbaarheid door 4 en door 25). Bewijs dat een natuurlijk getal deelbaar is door 4 als en slechts als het getal gevormd door de cijfers van de eenheden en tientallen deelbaar is door 4, en deelbaar is door 25 als en slechts als het getal gevormd door de cijfers van de eenheden en tientallen deelbaar is door 25.

U

Oefening 34 (kenmerk van deelbaarheid door 8 en door 125). Bewijs dat een natuurlijk getal deelbaar is door 8 als en slechts als het getal gevormd door de cijfers van de eenheden, tientallen en honderdtallen deelbaar is door 8, en deelbaar is door 125 als en slechts als het getal gevormd door de cijfers van de eenheden, tientallen en honderdtallen deelbaar is door 125.

U?

Oefening 35 (kenmerk van deelbaarheid door 3 en door 9). Bewijs dat een natuurlijk getal deelbaar is door 3 als en slechts als de som van zijn cijfers deelbaar is door 3, en deelbaar is door 9 als en slechts als de som van zijn cijfers deelbaar is door 9. Aanwijzing. Maak gebruik van Oefening 8 om aan te tonen dat 10n − 1 deelbaar is door 9 voor elke n ∈ N0 .

U??

Oefening 36 (lemma 2). Zij a, b ∈ Z. Bewijs dat er een geheel getal v is waarvoor aZ ∩ bZ = vZ.

U??

Oefening 37 (idealen van Z). Zij I een niet-lege deelverzameling van Z. Dan is I een ideaal van Z, notatie I C Z, als aan de volgende twee voorwaarden is voldaan: (1) voor alle r, s ∈ I geldt dat r − s ∈ I,

(2) voor alle k ∈ Z en r ∈ I geldt dat kr ∈ I.

Bewijs dat voor elke deelverzameling I ⊆ Z geldt: I CZ

∃d ∈ Z : I = dZ. XVI-18


Hoofdstuk 3

Priemgetallen De doelstelling van dit hoofdstuk is om de zogenaamde hoofdstelling van de rekenkunde aan te tonen: elk geheel getal kan geschreven worden als een product van priemgetallen, en dit op essentieel één manier.

3.1

Eenheden, priemgetallen en samengestelde getallen

In het vorige hoofdstuk hebben we aangetoond dat elk geheel getal a 6= 0 een even aantal delers heeft, meer bepaald: # del(a) ∈ {2, 4, 6, 8, . . .}. Gehele getallen met twee delers, zogenaamde eenheden, zijn eenvoudig te bepalen. Dat gebeurt in de eigenschap hieronder, geformuleerd als drie logisch gelijkwaardige uitspraken (i), (ii) en (iii): is één van hen waar dan zijn ook de andere waar, en is één van hen vals dan zijn de andere dat ook. Gelijkwaardige uitspraken worden ook wel equivalente uitspraken genoemd. 3 Eigenschap. Zij a ∈ Z. Dan zijn de volgende uitspraken equivalent:1 (i) # del(a) = 2, (ii) a = 1 of a = −1,

(iii) er bestaat een b ∈ Z zodat ab = 1. Zo’n geheel getal a wordt een eenheid van Z genoemd. De eenheden van Z zijn dus precies de getallen 1 en −1, en we noteren Z∗ = {1, −1}. Bewijstechniek. We moeten bewijzen dat (i), (ii) en (iii) equivalent zijn. Dit kunnen we doen door (i) ⇔ (ii), (ii) ⇔ (iii) en (iii) ⇔ (i) te bewijzen. Dit komt neer op zes deelbewijzen: (i)⇒(ii), (ii)⇒(i), (ii)⇒(iii) enzovoort. Het volstaat echter te bewijzen dat (i)⇒(ii), (ii)⇒(i) en (ii)⇔(iii), aangezien daar automatisch uit volgt dat (i)⇒(iii) en (iii)⇒(i). Bewijs van (i) ⇒ (ii). Stel dat # del(a) = 2. We moeten aantonen dat a = 1 of a = −1. We beweren dat a 6= 0.

Inderdaad, mocht a = 0 dan zou del(a) = Z0 . Een strijdigheid met # del(a) = 2. Welnu, 1 | a,

−1 | a,

a | a,

−a | a

wegens basiseigenschappen van deelbaarheid

⇒ 1, −1, a, −a ∈ del(a) ⇒

{1, −1, a, −a} ⊆ del(a)

⇒ #{1, −1, a, −a} ≤ # del(a) | {z } 2

zodat a = 1 of a = −1.

1 In de eerste plaats wordt elk geheel getal dat aan (iii) voldoet een eenheid van Z genoemd. Uit deze eigenschap volgt dan dat de eenheden van Z precies de gehele getallen 1 en −1 zijn.

XVI-19


Bewijs van (ii) ⇒ (i). Stel dat a = 1 of a = −1. We moeten aantonen dat # del(a) = 2. Welnu,

. als a = 1 dan is del(a) = {1, −1} zodat # del(a) = 2; . als a = −1 dan is del(a) = {1, −1} zodat # del(a) = 2.

Bewijs van (ii) ⇔ (iii). Er geldt: ∃b ∈ Z : ab = 1 ⇔ a | 1 ⇔

a ∈ del(1)

a ∈ {1, −1}

a = 1 of a = −1.

Gehele getallen met vier delers worden priemgetallen genoemd. Zo is bijvoorbeeld p = −7 een priemgetal, want del(−7) = {1, −1, 7, −7} zodat geldt: (i) # del(−7) = 4. Daarnaast merken we nog twee andere kenmerkende eigenschappen van het priegmetal −7 op. Is een product van twee gehele getallen gelijk aan −7, dan moet een van beide getallen ±1 zijn, in symbolen: (ii) ∀a, b ∈ Z : −7 = ab ⇒ a ∈ Z∗ of b ∈ Z∗ . Ten slotte merken we op: is het priemgetal −7 een deler van een product van twee gehele getallen, dan is noodzakelijk −7 en deler van (minstens) een van beide gehele getallen, in symbolen: (iii) ∀a, b ∈ Z : −7 | ab ⇒ −7 | a of − 7 | b. In de volgende stelling zullen we aantonen dat deze drie kenmerkende eigenschappen opgaan voor elk priemgetal p, en daarenboven ook gelijkwaardig zijn: is één van hen waar dan zijn de andere ook waar, en is één van hen vals dan zijn de andere dat ook. Stellen we bijvoorbeeld p = 6 dan geldt: (i) # del(6) = 4

is vals,

inderdaad, del(6) = {1, −1, 2, −2, 3, −3, 6, −6} zodat # del(6) 6= 4; (ii) ∀a, b ∈ Z : 6 = ab ⇒ a ∈ Z∗ of b ∈ Z∗

is vals,

inderdaad, is bijvoorbeeld a = 2 en b = 3 dan is 6 = ab en toch is a 6∈ Z∗ en b 6∈ Z∗ ; (iii) ∀a, b ∈ Z : 6 | ab ⇒ 6 | a of 6 | b

is vals,

inderdaad, is bijvoorbeeld a = −9 en b = 4 dan is 6 = ab en toch is 6 - a en 6 - b.

XVI-20


3 Stelling (eerste stelling van Euclides).2 Zij p ∈ Z0 en p 6∈ Z∗ . Dan zijn de volgende uitspraken equivalent: (i) # del(p) = 4, (ii) ∀a, b ∈ Z : p = ab ⇒ a ∈ Z∗ of b ∈ Z∗ ,

(iii) ∀a, b ∈ Z : p | ab ⇒ p | a of p | b.

Zo’n geheel getal p wordt een priemgetal van Z genoemd. Bewijstechniek. Het volstaat te bewijzen dat (i)⇔(ii) en (ii)⇔(iii) aangezien daar automatisch uit volgt dat (i)⇔(iii). Bewijs van (i) ⇒ (ii). Stel dat # del(p) = 4. Dan is del(p) = {1, −1, p, −p}. Inderdaad, omdat p 6= 0 is wegens basiseigenschappen van deelbaarheid: 1 | p, ⇒

−1 | p,

p | p,

Euclides van Alexandrië (derde eeuw voor Christus)

−p | p

{1, −1, p, −p} ⊆ del(p)

⇒ #{1, −1, p, −p} ≤ # del(p) . | {z } 4

Anderzijds is p 6∈ Z zodat #{1, −1, p, −p} = 4. We besluiten dat: ∗

#{1, −1, p, −p} ≤ # del(p) | {z } | {z } 4

4

zodat {1, −1, p, −p} = del(p).

We moeten aantonen dat ∀a, b ∈ Z : p = ab ⇒ a ∈ Z∗ of b ∈ Z∗ .

Neem a, b ∈ Z willekeurig en stel dat p = ab. We moeten aantonen dat a ∈ Z∗ of b ∈ Z∗ .

Welnu,

p = ab

a|p

a ∈ del(p)

a ∈ {1, −1, p, −p}

a = ±1 of a = ±p

a = ±1 of b = ±1

a ∈ Z∗ of b ∈ Z∗ .

want p = ab

Bewijs van (ii) ⇒ (i). Stel dat ∀a, b ∈ Z : p = ab ⇒ a ∈ Z∗ of b ∈ Z∗ . We moeten aantonen dat # del(p) = 4. We beweren dat del(p) = {1, −1, p, −p}.

Inderdaad, enerzijds is {1, −1, p, −p} ⊆ del(p) want omdat p 6= 0 is (basiseigenschappen van deelbaarheid): 1 | p,

−1 | p,

p | p,

−p | p

{1, −1, p, −p} ⊆ del(p)

en anderzijds is del(p) ⊆ {1, −1, p, −p} want neem x ∈ del(p) willekeurig, dan is p = xq voor een zekere q ∈ Z

⇒ x ∈ Z∗ of q ∈ Z∗

want ∀a, b ∈ Z : p = ab ⇒ a ∈ Z∗ of b ∈ Z∗

⇒ x = ±1 of q = ±1 ⇒ x = ±1 of x = ±p

want p = xq

⇒ x ∈ {1, −1, p, −p}. Dus # del(p) = #{1, −1, p, −p}. Nu is p 6∈ Z∗ zodat #{1, −1, p, −p} = 4. We besluiten dat # del(p) = 4. 2 In

feite wordt enkel de implicatie (i)⇒(iii) de eerste stelling van Euclides genoemd, naar Euclides, Elementen, Boek VII, Propositie 30. In de eerste plaats wordt elk geheel getal dat aan (ii) voldoet een irreducibel getal genoemd en elk geheel getal p dat aan (iii) voldoet een priemgetal genoemd. Uit deze stelling volgt dan dat de irreducibele getallen van Z precies de priemgetallen van Z zijn.

XVI-21


Bewijs van (iii) ⇒ (ii). Stel dat ∀a, b ∈ Z : p | ab ⇒ p | a of p | b.

We moeten aantonen dat ∀a0 , b0 ∈ Z : p = a0 b0 ⇒ a0 ∈ Z∗ of b0 ∈ Z∗ .

Neem a0 , b0 ∈ Z en stel dat p = a0 b0 . We moeten aantonen dat a0 ∈ Z∗ of b0 ∈ Z∗ .

Welnu, omdat p 6= 0 is wegens een basiseigenschap van deelbaarheid p|p

p | a0 b0

p | a0 of p | b0

want ∀a, b ∈ Z : p | ab ⇒ p | a of p | b.

We onderscheiden nu twee gevallen: . als p | a0 dan is

a0 = pq voor een zekere q ∈ Z ⇒ a0 b0 = pqb0 ⇒ p = pqb0

want p = a0 b0

⇒ 1 = qb0

wegens schrapwet voor de vermenigvuldiging

⇒ b0 ∈ Z∗

wegens een vorige eigenschap;

. als p | b dan is wegens een analoge redenering a0 ∈ Z∗ . 0

We besluiten dat a0 ∈ Z∗ of b0 ∈ Z∗ .

Bewijs van (ii) ⇒ (iii). Stel dat ∀a0 , b0 ∈ Z : p = a0 b0 ⇒ a0 ∈ Z∗ of b0 ∈ Z∗ . We moeten aantonen dat ∀a, b ∈ Z : p | ab ⇒ p | a of p | b.

Neem a, b ∈ Z en stel dat p | ab. We moeten aantonen dat p | a of p | b.

Nu zijn er maar twee gevallen mogelijk: p | a of p - a. Als p | a dan is het bewijs rond. Stel dus dat p - a. We moeten bewijzen dat p | b. Welnu, wegens een eerder lemma is

pZ + aZ = dZ We beweren dat d ∈ Z∗ .

voor een zekere d ∈ Z.

Inderdaad, we hebben dat p = p · 1 + a · 0 ∈ pZ + aZ ⇒ p ∈ dZ ⇒ p = du

voor een zekere u ∈ Z

⇒ d ∈ Z∗ of u ∈ Z∗

want ∀a0 , b0 ∈ Z : p = a0 b0 ⇒ a0 ∈ Z∗ of b0 ∈ Z∗ .

Mocht d 6∈ Z∗ dan zou u ∈ Z∗ zodat uit p = du volgt: d = ±p

een stijdigheid met het feit dat p - a.

dZ = pZ

a = p · 0 + a · 1 ∈ pZ + dZ = pZ

p|a

Omdat d ∈ Z∗ is dZ = Z zodat

pZ + aZ = Z

1 ∈ pZ + aZ

1 = pk + al

b = pbk + abl.

voor zekere k, l ∈ Z

Nu is volgt uit basiseigenschappen van deelbaarheid dat p | pbk en p | abl ⇒ p | (pbk + abl) ⇒ p | b.

XVI-22


De eerste positieve priemgetallen zijn gelijk aan 2, 3, 5, 7, 11, 13, 17, 19, 23, 29, 31, 37, 41, 43, 47, 53, . . . en uiteraard zijn de tegengestelde getallen ook priemgetallen: −2, −3, −5 enzovoort. Er is een eenvoudige manier om een tabel van (kleine) positieve priemgetallen op te stellen door middel van een procedure die bekend staat als de zeef van Eratosthenes. Stap 1. Schrijf alle natuurlijke getallen van 2 tot een met een zelf te kiezen maximum N op. Stap 2. Stel p = 2, het eerste positieve priemgetal. Stap 3. Verwijder alle veelvouden 2p, 3p, 4p, 5p, . . . uit de lijst. Eratosthenes van Cyrene (c.276 v.Chr. - c.195 v.Chr.)

Stap 4. Bepaal het eerste getal dat groter is dan p. (i) Als dit getal niet voorkomt in de (vernieuwde) lijst, dan stopt de procedure. (ii) Als dat getal wel voorkomt, dan is dat getal een priemgetal en noemen we dit nieuw getal p. Herhaal de procedure nu vanaf Stap 3. We illustreren deze werkwijze aan de hand van een voorbeeld. 3 Modelvoorbeeld. Bepaal alle positieve priemgetallen kleiner dan 100.

Oplossing. We schrijven alle natuurlijke getallen van 2 tot en met N = 100 op. Daarna passen we de stappen 2, 3 en 4 toe uit de procedure zeef van Eratosthenes (voer uit). 2

3

4

5

6

7

8

9

10

11

12

13

14

15

16

17

18

19

20

21

22

23

24

25

26

27

28

29

30

31

32

33

34

35

36

37

38

39

40

41

42

43

44

45

46

47

48

49

50

51

52

53

54

55

56

57

58

59

60

61

62

63

64

65

66

67

68

69

70

71

72

73

74

75

76

77

78

79

80

81

82

83

84

85

86

87

88

89

90

91

92

93

94

95

96

97

98

99

100

Op deze manier vinden we en vinden we alle positieve priemgetallen kleiner dan 100: 2, 43,

3,

5,

47,

7,

11,

13,

17,

19,

23,

29,

31,

37,

41,

53,

59,

61,

67,

71,

73,

79,

83,

89,

97.

De zeef van Eratosthenes doet ons vermoeden dat er oneindig veel priemgetallen bestaan. Om dat resultaat aan te tonen, moeten we ons wenden tot getallen met meer dan vier delers. Gemakshalve geven we die een naam. 3 Definitie. Zij a ∈ Z0 . We noemen a een samengesteld getal als # del(a) > 4. Voorbeeld. De getallen 15, −6 en 918 zijn samengesteld. De getallen −7, 1 en 0 zijn niet samengesteld. XVI-23


3 Lemma. Elk samengesteld getal is deelbaar door een priemgetal. Zo’n priemgetal wordt een priemdeler van dat samengesteld getal genoemd. Bewijs. Zij a een samengesteld getal, dus a ∈ Z0 en # del(a) > 4. Dan is a 6∈ Z∗ en {1, −1, a, −a} $ del(a). Welnu, wegens het principe van de welordening bevat del(a) een kleinste positief geheel getal groter dan 1. Noem dat geheel getal p. We beweren dat p een priemgetal is. Inderdaad, mocht p geen priemgetal zijn dan zou # del(p) > 4. Dan is {1, −1, p, −p} $ del(p). wegens het principe van de welordening bevat del(p) een kleinste positief geheel getal groter dan 1. Noem dat geheel getal q. Dan is: q | p en p | a ⇒ q | a zodat q ∈ del(a) en 1 < q < p. Dit is in strijd met het feit dat p het kleinste positief geheel getal in del(a) groter dan 1 is.

3 Stelling (tweede stelling van Euclides)3 . Er zijn oneindig veel priemgetallen. Bewijstechniek. Een bewijs uit het ongerijmde is een bewijstechniek waarbij we het tegenovergestelde aannemen van wat we willen bewijzen en dan trachten een contradictie (of tegenstrijdigheid) te verkrijgen. Aangezien tegenstrijdigheden binnen de wiskunde onmogelijk zijn, moet die tegenovergestelde aanname foutief geweest zijn. Hiermee is de te bewijzen stelling aangetoond. Bewijs van Euclides. Veronderstel, uit het ongerijmde, dat er een eindig aantal priemgetallen zijn. 2, 3, 5, 7, 11, . . . , p alle positieve priemgetallen en beschouw het natuurlijk getal

Noem

n = 2 · 3 · 5 · 7 · 11 . . . p + 1. Dan is n > p zodat n geen priemgetal is. Dus n is een samengesteld getal. Wegens het vorige lemma heeft n een positieve priemdeler q. Welnu, omdat 2, 3, 5, 7, 11, . . . , p alle positieve priemgetallen zijn is dus: q ∈ {2, 3, 5, . . . , p} ⇒ q | 2 · 3 · 5 · 7 · 11 . . . p ⇒ q | (n − 2 · 3 · 5 · 7 · 11 . . . p) ⇒ q|1 ⇒ q = ±1. Dit is in strijd met het feit dat q een priemgetal is. We besluiten dat er oneindig veel priemgetallen zijn.

Het is een misvatting om te denken dat elk natuurlijk getal n = 2 · 3 · 5 · 7 · 11 . . . p + 1 met p een positief priemgetal, steeds zelf een priemgetal is. Inderdaad: 2·3+1=7

priemgetal

2 · 3 · 5 + 1 = 31 2 · 3 · 5 · 7 + 1 = 211

priemgetal priemgetal

2 · 3 · 5 · 7 · 11 + 1 = 2311

priemgetal

2 · 3 · 5 · 7 · 11 · 13 + 1 = 30 031 = 59 · 509

geen priemgetal.

3 Euclides, Elementen, Boek IX, Propositie 20. Net zoals vele stellingen in de wiskunde, zijn er meerdere bewijzen ontdekt om dit resultaat aan te tonen. Zes bijzondere elegante bewijzen zijn terug te vinden in [1, Chapter 1], waaronder het bewijs van Euclides zoals hierboven vermeld en het bewijs van Hurtwitz dat in Hoofdstuk 4 aan bod komt. In 2015 bedacht Sam Northshield [9] een manier om de redenering uit het bewijs van Euclides in één regel uit te drukken. Voor een meer elementair eenregelbewijs verwijzen we naar Oefening 9.

XVI-24


Door een samengesteld getal a te delen door een echte deler van a, kunnen we a schrijven als een product van twee gehele getallen die elk minder delers dan a bevatten. Door deze procedure herhaaldelijk toe te passen, kunnen we a schrijven als een product van niet-samengestelde getallen. Zo is bijvoorbeeld: 918 = 2 · 459 = 2 · 3 · 153 = 2 · 3 · 3 · 51 = 2 · 3 · 3 · 3 · 17 = 2 · 33 · 17. Het algemeen resultaat zullen we hieronder formeel bewijzen. 3 Stelling. Elk samengesteld getal is een product van priemgetallen. Zo’n product van priemgetallen wordt een ontbinding van dat samengesteld getal genoemd. Bewijs. Het volstaat om aan te tonen dat elk positief samengesteld getal een product van priemgetallen is. Zij a een positief samengesteld getal. Wegens een voorgaand lemma heeft a een positieve priemdeler p1 , zodat: a = p1 · a1

voor een zekere a1 ∈ Z met 1 < a1 < a.

We onderscheiden nu twee gevallen. . Als a1 een priemgetal is, dan is a een product van priemgetallen en is het bewijs klaar. . Als a1 geen priemgetal is, dan is a1 een positief samengesteld getal. Wegens een voorgaand lemma heeft a1 een positieve priemdeler p2 , zodat: a = p1 · p2 · a2

voor een zekere a2 ∈ Z met 1 < a2 < a1 < a.

We onderscheiden nu twee gevallen. Als a2 een priemgetal is, dan is a een product van priemgetallen en is het bewijs klaar. Als a2 geen priemgetal is, dan is a2 een positief samengesteld getal. Wegens een voorgaand lemma . . . Blijven we deze procedure herhalen, dan verkrijgen we een strikt dalende rij a, a1 , a2 , a3 , . . . waarbij elke term een natuurlijk getal groter dan 1 is: 1 < . . . < a3 < a2 < a1 < a. Dus deze procedure stopt na een eindig aantal stappen, zodat a een product van een eindig aantal priemgetallen is.

XVI-25


3.2

Hoofdstelling van de rekenkunde

Een samengesteld getal kan op meerdere manieren als een product van priemgetallen geschreven worden. Zo is bijvoorbeeld: −68 = (−2) · 2 · 17 = 22 · (−17) = 2 · (−17) · 2 = 17 · 2 · (−2). Door een eenheid voorop te schrijven, kunnen we ervoor zorgen dat alle priemgetallen die in zo’n product voorkomen positief zijn. Door deze priemgetallen te rangschikken van klein naar groot, vinden we slechts één ontbinding: −68 = (−1) · 22 · 17. Nu geldt dat er voor elk samengesteld getal slechts één zo’n ontbinding is. 3 Stelling (hoofdstelling van de rekenkunde).4 Zij a ∈ Z0 . Dan kan a op precies één manier geschreven worden als a = e pa1 1 pa2 2 . . . pakk waarbij e ∈ Z∗ , k ∈ N, ai ∈ N0 en p1 < p2 < . . . < pk positieve priemgetallen zijn. Deze unieke schrijfwijze noemen we de standaardontbinding van a. Bewijs. Als a een eenheid of een priemgetal is, dan zijn we klaar. Inderdaad, als a een eenheid is, stel dan e = a en k = 0. En als a een priemgetal is, stel dan e = sign(a), k = 1, p1 = |a| en a1 = 1. Dus voor het vervolg van het bewijs mogen we aannemen dat a een samengesteld getal is.

Carl Friedrich Gauss (1777-1855)

Wegens de vorige stelling kan a geschreven worden als een product van priemgetallen. Door eventuele mintekens voorop te plaatsen en factoren te herschikken, kunnen we a alvast op minstens één manier schrijven als a = e pa1 1 pa2 2 . . . pakk

(∗)

waarbij e ∈ Z∗ , k ∈ N, ai ∈ N0 en p1 < p2 < . . . < pk positieve priemgetallen zijn.

Om aan te tonen dat er hoogstens één manier is, veronderstellen we dat er nog een tweede manier is: a = e0 q1b1 q2b2 . . . qlbl

(∗∗)

waarbij e0 ∈ Z∗ , l ∈ N, bj ∈ N0 en q1 < q2 < . . . < ql positieve priemgetallen zijn. We moeten aantonen: e = e0 ,

k = l,

pi = qi voor elke i

en

ai = bi voor elke i.

Welnu, uit (∗) en (∗∗) volgt: e pa1 1 pa2 2 . . . pakk = e0 q1b1 q2b2 . . . qlbl .

(∗ ∗ ∗)

(1) We beweren dat e = e0 . Inderdaad: e, e0 ∈ Z∗ en alle pi , qj zijn positief, dus e = e0 . Omdat e = e0 volgt uit (∗ ∗ ∗):

pa1 1 pa2 2 . . . pakk = q1b1 q2b2 . . . qlbl .

4 Blijkbaar

(∗ ∗ ∗ ∗)

werd de hoofdstelling van de rekenkunde pas voor het eerst expliciet vermeld door Gauss in 1798, wellicht omdat hij de eerste was die het belang hiervan inzag. Uiteraard was dit resultaat bekend bij eerdere wiskundigen, maar Gauss was de eerste die getaltheorie op een systematische manier ontwikkeld heeft.

XVI-26


(2) We beweren dat k = l en pi = qi voor elke i. Inderdaad, in (∗ ∗ ∗ ∗) is: p1 | LL

p1 | RL

p1 | qj voor een zekere j

qj = p1 · x voor een zekere x ∈ Z

qj = p1

want eerste stelling van Euclides (iii) p1 | · 4 ⇒ p1 | of p1 | 4 want eerste stelling van Euclides (ii) qj = · 4 ⇒ ∈ Z∗ of 4 ∈ Z∗ en p1 > 0, qj > 0

⇒ p1 komt voor in RL. Passen we deze redenering voor p1 nu ook toe voor de priemgetallen p2 , . . . , pk dan verkrijgen we: p1 , p2 , . . . , pk komt voor in RL. Een analoge redenering geeft: q1 , q2 , . . . , ql komt voor in LL. Omdat p1 < p2 < . . . < pk en q1 < q2 < . . . < ql volgt hieruit dat k = l en pi = qi voor elke i.

Omdat k = l en pi = qi voor elke i volgt uit (∗ ∗ ∗ ∗): pa1 1 pa2 2 . . . pakk = pb11 pb22 . . . pbkk .

(3) We beweren dat ai = bi voor elke i. Inderdaad, mocht bijvoorbeeld a1 > b1 dan is wegens de schrapwet: pa1 1 pa2 2 . . . pakk = pb11 pb22 . . . pbkk ⇒ pa1 1 −b1 pa2 2 . . . pakk = pb22 . . . pbkk

(∗ ∗ ∗ ∗ ∗)

Nu is a1 > b1

a1 − b1 > 0

p1 | LL van (∗ ∗ ∗ ∗ ∗)

p1 | RL van (∗ ∗ ∗ ∗ ∗)

p1 | p2 of p1 | p3 of . . . of p1 | pk

want eerste stelling van Euclides (iii)

p1 = p2 of p1 = p3 of . . . of p1 = pk

want eerste stelling van Euclides (ii).

Dit is in strijd met p1 < p2 < . . . < pk . Dus a1 ≤ b1 . Een analoge redenering geeft dat b1 ≤ a1 , dus is a1 = b1 . Een analoge redenering geeft dat a2 = b2 , . . . , ak = bk .

XVI-27


Hoewel er verschillende technieken zijn om een geheel getal te ontbinden in priemfactoren, is er tot op heden geen efficiënt algoritme bekend. Net die onwetendheid garandeerd de veiligheid van heel wat hedendaagse codeermechanismen, waarvan vele afgeleid zijn van de codeermethode RSA.5 Natuurlijke getallen √ n die relatief klein zijn kunnen handmatig ontbonden worden met behulp van een lijst van priemgetallen tot en met n. Inderdaad: elk samengesteld getal n heeft een kleinste positieve priemdeler p1 , en uit de standaardontbinding van n volgt eenvoudig dat p21 ≤ n. 3 Modelvoorbeeld. Bepaal de standaardonbinding van 5566. Maak gebruik van je grafische rekenmachine. Oplossing. Het getal 5566 is even, zodat het deelbaar is door 2. Zo vinden √ we al dat 5566 = 2 · 2783. We gaan na of 2783 deelbaar is door een positief priemgetal kleiner of gelijk aan 2783 = 52, 7 . . .. Zo niet, dan is 2783 zelf een priemgetal. Met behulp van de zeef van Eratosthenes vinden we de positieve priemgetallen kleiner dan 53: 2, 3, 5, 7, 11, 13, 17, 19, 23, 29, 31, 37, 41, 43, 47. Ter illustratie vervolgen we de oefening zonder gebruik te maken van de grafische rekenmachine. Wegens kenmerken van deelbaarheid is: 2 - 2783 want het laatste cijfer van 2783 is 3, en 3 is niet deelbaar door 2 3 - 2783 want de som van de cijfers van 2783 is 20, en 20 is niet deelbaar door 3 5 - 2783 want het laatste cijfer van 2783 is 3, en 3 is niet deelbaar door 5 7 - 2783 want trek het laatste cijfer tweemaal af van het getal gevormd door de overige cijfers: 7 | 2783

7 | (278 − 2 · 3)

7 | 2783

7 | 272

7 | 2783

7 | (27 − 2 · 2)

7 | 2783

7 | 23

NEE!

11 | 2783 want trek het laatste cijfer eenmaal af van het getal gevormd door de overige cijfers: 11 | 2783

11 | (278 − 3)

11 | 2783

11 | 275

11 | 2783

11 | (27 − 5)

11 | 2783

11 | 22

JA!

Door middel van een staartdeling vinden we 2783 = 11 · 253, dus 5566 = 2 · 2783 = 2 · 11 · 253. √ √ We herhalen dit proces met het deeltal 253. Omdat 253 < 256 = 16 en we deelbaarheid door de positieve priemgetallen kleiner dan 11 al nagegaan hebben, hoeven we nu enkel de priemgetallen 11 en 13 te controleren. Zo vinden we uiteindelijk de standaardontbinding van 5566: 5566 = 2 · 11 · 253 = 2 · 11 · 11 · 23 = 2 · 112 · 23.

5 Het

formele algoritme werd in 1977 ontworpen door Ron Rivest, Adi Shamir en Len Adleman (vandaar de afkorting RSA), zie [10].

XVI-28


De hoofdstelling van de rekenkunde leidt tot een krachtig criterium voor deelbaarheid van twee gehele getallen. Zo is bijvoorbeeld a = 57 · 193 · 372 deelbaar door b = 53 · 372 omdat elke priemdeler van b ook een priemdeler van a is en omdat de exponenten van de priemdelers van b kleiner of gelijk zijn aan de exponenten van de priemdelers van a. Om ook de priemdeler 19 van a in rekening te brengen, is het handig om de standaardontbindingen van a en b te herschrijven als: a = 57 · 193 · 372

b = 53 · 190 · 372 .

en

Zo kunnen voor elk tweetal gehele getallen verschillend van nul de standaardontbinding herschrijven met dezelfde priemgetallen. Als bijvoorbeeld a = 3 · 52 en b = 2 · 73 dan kunnen we dit schrijven als: a = 20 · 3 · 52 · 70

b = 2 · 30 · 50 · 73 .

en

Zo hoeven we niet steeds bij te houden welke priemfactoren wel of niet in a en b bevat zijn. 3 Gevolg. Zij a, b ∈ Z0 en stel a = e pa1 1 pa2 2 · · · pakk

en

b = e0 pb11 pb22 · · · pbkk

waarbij e, e0 ∈ Z∗ , k ∈ N, ai , bi ∈ N en p1 < p2 < . . . < pk positieve priemgetallen zijn. Dan geldt: b|a

bi ≤ ai voor alle i

Bewijs. Alvast is e pa1 1 pa2 2 · · · pakk a = = e00 p1a1 −b1 pa2 2 −b2 · · · pakk −bk ∈ Q b e0 pb11 pb22 · · · pbkk waarbij e00 ∈ Z∗ .

Stel eerst dat bi ≤ ai voor alle i. Dan geldt: a 1 − b1 ≥ 0

,

a2 − b2 ≥ 0

pa1 1 −b1 ∈ Z ,

pa1 1 −b1 pa2 2 −b2 · · · pakk −bk ∈ Z

a ∈Z b

pa2 2 −b2 ∈ Z ,

,

... ...

a k − bk ≥ 0

, ,

pakk −bk ∈ Z

⇒ b | a.

Omgekeerd, stel dat b | a. Dan is a = bq voor een zeker geheel getal q, dus e pa1 1 pa2 2 · · · pakk = e0 pb11 pb22 · · · pbkk q. Mocht bijvoorbeeld b1 > a1 dan zou e pa2 2 · · · pakk = e0 pb11 −a1 pb22 · · · pbkk q. Maar dan leidt LL tot een standaardontbinding die het priemgetal p1 niet bevat, terwijl het RL leidt tot een standaardontbinding die het priemgetal p1 wel bevat. Dit is strijdig met de hoofdstelling van de rekenkunde. We besluiten dat b1 ≤ a1 . Analoog bewijst men dat bi ≤ ai voor alle i.

XVI-29


Oefeningen 3 Priemgetallen

Basis ?

Verdieping ? ??

??

3.1 Eenheden, priemgetallen, samengestelde getallen

1 2

3 4 5

6

7 8

3.2 Hoofdstelling van de rekenkunde

11 12

13 14

15

16

Uitbreiding ? ?? 9 10 17 18 19

20

21

Oefeningen bij §3.1 B

Oefening 1. Geef de zeven kleinste natuurlijke getallen die samengesteld zijn.

B

Oefening 2. Zij a, p ∈ Z waarbij p een priemgetal is. Bepaal alle gehele getallen n die een deler zijn van an + p.

B?

Oefening 3. Bepaal alle gehele getallen a en b waarvoor a2 − b2 = 79.

B?

Oefening 4. Bepaal telkens alle natuurlijke getallen n waarvoor p een priemgetal is. (a) p = 17n

(c) p = n2 − 4

(b) p = n2 − 1

(d) p = n3 + 1

B?

Oefening 5. Bewijs dat er oneindig veel samengestelde getallen bestaan.

V

Oefening 6 (Vlaamse Wiskunde Olympiade 1990 eerste ronde). Definieer n! = 1·2·3 · · · (n−1)·n, het produkt van de natuurlijke getallen van 1 tot en met n. Het aantal priemgetallen p met eigenschap 77! + 1 < p < 77! + 77 is gelijk aan (A) 0

(B) 1

(C) 7

(D) 11

(E) 17

V?

Oefening 7. Zij n ∈ N. Bewijs dat het verschil van twee opeenvolgende n-de machten altijd een oneven getal is.

V?

Oefening 8. Zij d ∈ N willekeurig. Bewijs dat er een geheel getal bestaat die precies 2d echte delers heeft.

U?

6 Oefening 9 (karakteristieke functies en een eenregelbewijs voor de tweede ß stelling van Euclides). 1 als x ∈ A Is A ⊆ R dan wordt de karakteristieke functie χA van A gedefinieerd als χA (x) = 0 als x ∈ R \ A. Verklaar elke stap in het volgende eenregelbewijs van de tweede stelling van Euclides: mocht de verzameling P van alle priemgetallen eindig zijn, dan zou Ç å Q Å ã X X 1 + p0 ∈P p0 1 χ χ = > 0. 0= Z Z p p p∈P

p∈P

U?

Oefening 10 (bovengrens voor positieve priemdelers in samengestelde getallen). Zij n een positief samengesteld getal en noem p de kleinst positieve deler van n verschillend van 1. Bewijs: (a) p is een priemgetal, √ (b) p ≤ n. 6 De inspiratie voor dit bewijs van de tweede stelling van Euclides werd gehaald uit het origineel eenregelbewijs van Sam Northshield uit 2015 [9], waarbij gesteund wordt op de periodiciteit van de sinusfunctie (zie Deel Precalculus 2):

0<

Y p∈P

sin

π p

=

Y

Å

π(1 +

sin

p∈P

XVI-30

Q

p0 ∈P

p

p0 )

ã = 0.


Oefeningen bij §3.2 B

Oefening 11. Bepaal het grootste negatief geheel getal dat precies vijf verschillende priemdelers heeft.

B

Oefening 12. Bepaal het kleinste positief geheel getal n waarvoor 1260n de derdemacht van een natuurlijk getal is.

B?

Oefening 13. Bepaal telkens de standaardontbinding van het geheel getal. Maak gebruik van je grafische rekenmachine.

B?

(a) −899

(c) −5820

(b) 1485

(d) 9075

Oefening 14 (Vlaamse Wiskunde Olympiade 1986 eerste ronde). Hoeveel verschillende positieve delers heeft het getal 30 030 = 2 · 3 · 5 · 7 · 11 · 13? (A) 6

(B) 36

(C) 62

(D) 64

(E) 128

V

Oefening 15. Bepaal de rest bij deling van 2015! door 2016.

V?

Oefening 16. Bepaal 100 opeenvolgende gehele getallen waarvan geen enkele een priemgetal is.

U

Oefening 17 (perfecte getallen). Een natuurlijk getal wordt perfect (of volmaakt) genoemd als het gelijk is aan de som van zijn echte positieve delers. Toon aan dat 6 en 28 perfecte getallen zijn.7

U

Oefening 18 (gehele getallen met zes delers). Toon aan dat de gehele getallen met zes verschillende delers gegeven worden door ±p2 waarbij p een priemgetal is.

U

Oefening 19 (bevriende getallen). Van twee verschillende natuurlijke getallen a, b wordt gezegd dat ze bevriend zijn als de som van de echte positieve delers van a gelijk is aan b, en de som van de echte positieve delers van b gelijk is aan a. Toon aan dat 220 en 284 bevriende getallen zijn.8

U?

Oefening 20 (gehele getallen met acht delers). Toon aan dat de gehele getallen met acht verschillende delers gegeven worden door ±p3 en ± pq waarbij p, q verschillende priemgetallen zijn. Aanwijzing. Maak gebruik van Oefening 21(a).

U??

Oefening 21 (aantal positieve delers van een natuurlijk getal). Zij n ∈ N0 en stel dat de standaardontbinding van n gegeven wordt door n = pn1 1 pn2 2 . . . pnk k waarbij k ∈ N, ni ∈ N0 en p1 < p2 < . . . < pk positieve priemgetallen zijn. (a) Toon aan dat het aantal positieve delers van n gegeven wordt door de formule d(n) =

k Y

(ni + 1).

i=1

(b) Bepaal het kleinste natuurlijke getal dat 15 positieve delers heeft. 7 Euclides van Alexandrië bewees dat als 2p − 1 een priemgetal is, dat dan 2p−1 (2p − 1) een perfect getal is. In 1747 bewees Leonhard Euler dat elk even perfect getal van die vorm is. Het is tot op heden niet bekend of er een perfect getal bestaat dat oneven is. 8 De eerste vijf paar paren bevriende getallen zijn (220, 284), (1184, 1210), (2620, 2924), (5020, 5564) en (6232, 6368). Leonhard Euler vond tussen 1747 en 1750 een zestigtal nieuwe bevriendegetallenparen. Anno 2007 waren er bijna twaalf miljoen paren bevriende getallen bekend. Het is niet bekend of er oneindig veel paren bevriende getallen zijn, en of er bevriende getallen bestaan die onderling ondeelbaar zijn.

XVI-31


De afgelopen jaren is gelijke kansen te zeer herleid tot “iedereen moet gelijk behandeld worden”. Zo gaan we naar de kleinst gemene deler. Een samenleving heeft absoluut nood aan mensen die schitteren. Pascal Smet, Vlaams minister van Onderwijs 2009-2014 De Standaard, 29/08/2009

Hoofdstuk 4

Grootste gemene delers en kleinste gemene veelvouden In dit hoofdstuk bestuderen we enkele bijzondere getallen: grootste gemene delers, kleinste gemene veelvouden, Fermatgetallen en Mersennegetallen. Tot slot passen we deelbaarheid van gehele getallen toe op tandwielen.

4.1

Grootste gemene delers

We hernemen een eerder voorbeeld: −12Z + 8Z = 4Z. Merk op dat ook −12Z + 8Z = (−4)Z. Niet toevallig zijn 4 en −4 zogenaamde grootste gemene delers van −12 en 8, hetgeen betekent:1 (1) 4 en −4 zijn beide een deler van −12 en 8, en

(2) 4 en −4 zijn beide een veelvoud van elke andere deler c van −12 en 8. In het algemeen worden de grootste gemene delers van twee gehele getallen a en b als volgt gedefinieerd. 3 Definitie. Zij a, b ∈ Z niet beide nul. Een grootste gemene deler van a en b is een geheel getal d 6= 0 dat voldoet aan de volgende twee voorwaarden: (1) d | a en d | b, (2) ∀c ∈ Z0 : c | a en c | b ⇒ c | d. Is 1 een grootste gemene deler van a en b, dan noemen we a en b relatief priem. Voorbeeld. Een grootste gemene deler van −18 en 24 is −6. Een grootste gemene deler van 108 en 0 is 108. De getallen 38 en −7 zijn relatief priem.

3 Eigenschap 1. Zij a, b ∈ Z niet beide nul en d ∈ Z0 . Dan zijn de volgende uitspraken equivalent: (i) d is een grootste gemene deler van a en b, (ii) dZ = aZ + bZ.

Bewijs van (i) ⇒ (ii). Stel dat d een grootste gemene deler van a en b is. Wegens een eerder lemma is aZ + bZ = d0 Z

voor een zekere d0 ∈ Z.

We beweren dat d | d0 .

d0 = d0 · 1 ∈ d0 Z

Inderdaad, ⇒

d0 ∈ aZ + bZ

⇒ d0 = ak + bl

voor zekere k, l ∈ Z.

Nu is d een grootste gemene deler van a en b zodat d|a

en

⇒ d | (ak + bl)

d|b

wegens de definitie van grootste gemene deler, deel (1) wegens een basiseigenschap van deelbaarheid

⇒ d | d0 . 1 In deze context is de term grootste eerder misleidend, daar bijvoorbeeld −4 als een grootste gemene deler van −12 en 8 net kleiner dan of gelijk aan elke gemeenschappelijke gehele deler van −12 en 8 is. Het begrip grootste moet hier dus opgevat worden als grootste in absolute waarde, daar het wel zo is dat |−4| groter dan of gelijk aan (de absolute waarde van) elke gemeenschappelijke gehele deler van −12 en 8 is.

XVI-32


Daarnaast beweren we dat ook d0 | d. a = a · 1 + b · 0 ∈ aZ + bZ

Inderdaad,

⇒ a ∈ d0 Z ⇒ a = d0 q

voor een zekere q ∈ Z

⇒ d0 | a en een analoge redenering levert dat d0 | b. Hierbij hebben we gesteund op het feit dat d0 6= 0. Dat is inderdaad zo, want mocht d0 = 0 dan zou aZ + bZ = {0}, en omdat a2 + b2 = a · a + b · b ∈ aZ + bZ zou dan a2 + b2 = 0, waaruit zou volgen dat a = 0 en b = 0, in strijd met het gegeven. Nu is d een grootste gemene deler van a en b zodat ∀c ∈ Z0 : c | a en c | b ⇒ c | d

wegens de definitie van grootste gemene deler, deel (2).

Omdat d0 | a en d0 | b volgt hier dus uit dat d0 | d.

Uit deze twee beweringen volgt nu: d | d0

en

d0 | d

⇒ d0 = ±d ⇒ d0 Z = dZ ⇒ aZ + bZ = dZ. Bewijs van (ii) ⇒ (i). Stel dat aZ+bZ = dZ. We moeten aantonen dat d een grootste gemene deler van a en b is. Alvast is d 6= 0 (zie gegeven). Dus het volstaat om aan te tonen dat d voldoet aan de twee voorwaarden uit de definitie van grootste gemene deler: (1) d | a en d | b, (2) ∀c ∈ Z0 : c | a en c | b ⇒ c | d. Welnu, a = a · 1 + b · 0 = aZ + bZ

(1) We hebben dat

⇒ a ∈ dZ ⇒ a = dq 0 ⇒ d|a

voor een zekere q 0 ∈ Z want d 6= 0, zie gegeven

en analoog is d | b. (2) Neem c ∈ Z0 en stel dat c | a en c | b. We moeten aantonen dat c | d. Welnu,

d = d · 1 ∈ dZ ⇒ d ∈ aZ + bZ ⇒ d = ak 0 + bl0

Nu is

c|a

en

voor zekere k 0 , l0 ∈ Z.

c|b

⇒ c | (ak 0 + bl0 )

wegens een basiseigenschap van deelbaarheid

⇒ c | d.

XVI-33


Een belangrijk gevolg van deze eigenschap is dat elk tweetal gehele getallen a, b niet beide nul precies twee grootste gemene delers hebben, waarvan de ene tegengesteld is aan de andere. De positieve grootste gemene deler van a en b zullen we met ggd(a, b) noteren. Merk dus op dat: ggd(a, b)Z = aZ + bZ = (− ggd(a, b))Z. Voorgaande resultaten leiden nu tot de zogenaamde stelling van Bachet-Bézout.2 3 Stelling (Bachet-Bézout). Zij a, b ∈ Z niet beide nul en zij d = ggd(a, b) de positieve grootste gemene deler van a en b. Dan geldt: (1) er bestaan gehele getallen k, l waarvoor d = ak + bl, (2) d is het kleinste strikt positief geheel getal dat een lineaire combinatie van a en b is, (3) elke lineaire combinatie van a en b is een veelvoud van d. Bewijs van (1). Wegens de vorige eigenschap is dZ = aZ + bZ. Welnu, d = d · 1 ∈ dZ ⇒ d ∈ aZ + bZ ⇒ d = ak + bl

Claude Gaspard Bachet de Méziriac (1581 - 1638)

voor zekere k, l ∈ Z.

Bewijs van (2). Alvast volgt uit (1) dat d een lineaire combinatie van a en b is. Mocht d niet het kleinste strikt positief geheel getal zijn dat een lineaire combinatie van a en b is, dan zou er een d0 ∈ Z bestaan met (1) 0 < d0 < d

en

(2) d0 = ak 0 + bl0

voor zekere k 0 , l0 ∈ Z.

Welnu, uitspraak (2) betekent: d0 = ak 0 + bl0

voor zekere k 0 , l0 ∈ Z

⇒ d0 ∈ aZ + bZ ⇒ d0 ∈ dZ ⇒ d0 = dq

voor een zekere q ∈ Z.

Maar dan leidt uitspraak (1) tot: 0 < d0 < d ⇒

0 < dq < d

0 < q < 1,

hetgeen in strijd is met het feit dat q ∈ Z. Bewijs van (3). Stel dat x een lineaire combinatie van a en b is. We moeten aantonen dat x een veelvoud van d is. Welnu, x = ak 00 + bl00

2 Voor

voor zekere k 00 , l00 ∈ Z

x ∈ aZ + bZ

x ∈ dZ

x is een veelvoud van d.

het eerst aangetoond door Bachet in 1624 [2]. In 1779 bewees Étienne Bézout

XVI-34

het gelijkaardig resultaat voor veeltermen [3].


De schrijfwijze van positieve grootste gemene deler als een lineaire combinatie van a en b is niet uniek. Zo is bijvoorbeeld ggd(6, 4) = 2, waarbij 6 · 1 + 4 · (−1) = 2 maar ook 6 · (−3) + 4 · 5 = 2.

Kennen we van a, b ∈ Z0 de standaardontbinding, dan kunnen we ggd(a, b) op een eenvoudige manier vinden. Beschouw bijvoorbeeld a = 36 en b = 120. Dan is ggd(a, b) = ggd(36, 120) = 12. Nu kunnen we de standaardontbindingen van a, b en ggd(a, b) herschrijven als a = 36 = 22 · 32 · 50 b = 120 = 23 · 31 · 51 ggd(a, b) = 12 = 22 · 31 · 50 waarmee we inzien dat de volgende formule geldt: ggd(a, b) = 2min(2,3) · 3min(2,1) · 5min(0,1) . Deze formule is eenvoudig te veralgemenen. 3 Eigenschap 2. Zij a, b ∈ Z0 en stel a = e pa1 1 pa2 2 · · · pakk en b = e0 pb11 pb22 · · · pbkk waarbij e, e0 ∈ Z∗ , k ∈ N, ai , bi ∈ N en p1 < p2 < . . . < pk positieve priemgetallen zijn. Dan geldt: min(a1 ,b1 ) min(a2 ,b2 ) p2

ggd(a, b) = p1

min(ak ,bk )

. . . pk

Bewijs. Beschouw het geheel getal min(a1 ,b1 ) min(a2 ,b2 ) p2

d = p1

min(ak ,bk )

. . . pk

.

Aan te tonen: d = ggd(a, b). Alvast is d > 0. Dus het volstaat om aan te tonen dat d voldoet aan de twee voorwaarden uit de definitie van grootste gemene deler: (1) d | a en d | b,

(2) ∀c ∈ Z0 : c | a en c | b ⇒ c | d. Welnu, (1) wegens een eerder gevolg is d|a

min(ai , bi ) ≤ ai voor alle i {z } | OK!

zodat we mogen besluiten dat d | a. Analoog is d | b. (2) Neem c ∈ Z0 en stel dat c | a en c | b. Aan te tonen c | d. Welnu, omdat c | a is wegens een eerder gevolg c = e00 pc11 pc22 · · · pckk

voor zekere e00 ∈ Z∗ en ci ∈ N

en waarbij ci ≤ ai voor elke i. Omdat c | b is ook ci ≤ bi voor elke i. Nu is c|d

ci ≤ min(ai , bi ) voor alle i {z } | OK!

zodat inderdaad c | d.

XVI-35


Kennen we van twee gehele getallen hun standaardontbinding, dan kunnen we hun positieve grootste gemene deler eenvoudig berekenen door Eigenschap 2 toe te passen. 3 Modelvoorbeeld 1. Bepaal de positieve grootste gemene deler van de gehele getallen a = 11 016 = 23 · 34 · 17

en b = −21 888 = (−1) · 27 · 32 · 19.

Oplossing. We kunnen de standaardontbindingen van a en b herschrijven als: a = 11 016 = 23 · 34 · 171 · 190

en

b = −21 888 = (−1) · 27 · 32 · 170 · 191

zodat wegens Eigenschap 2: ggd(a, b) = 2min(3,7) · 3min(4,2) · 17min(1,0) · 19min(0,1) = 23 · 32 · 170 · 190 =8·9 = 72.

Eigenschap 2 kan ook aangewend worden om basiseigenschappen van grootste gemene delers te bewijzen. 3 Modelvoorbeeld 2. Zij a, b, c ∈ Z0 en stel dat ggd(a, b) = 1 en c | a. Bewijs dat ggd(b, c) = 1. Oplossing. We kunnen de standaardontbinding van a, b en c herschrijven als a = epa1 1 pa2 2 . . . pakk

en

b = e0 pb11 pb22 . . . pbkk

en

c = e00 pc11 pc22 . . . pckk

waarbij e, e0 , e00 ∈ Z∗ , k ∈ N, ai , bi , ci ∈ N en p1 < p2 < . . . < pk positieve priemgetallen zijn.

We herformuleren eerst de gegevens. Passen we Eigenschap 2 toe op het eerste gegeven, dan verkrijgen we: ggd(a, b) = 1 ⇒

min(ai , bi ) = 0 voor elke i

⇒ ai = 0 of bi = 0 voor elke i

(∗)

terwijl het tweede gegeven leidt tot: c|a

ci ≤ ai

voor elke i.

Dankzij Eigenschap 2 kunnen we nu ook het te bewijzen herformuleren: ggd(b, c) = 1

min(bi , ci ) = 0 voor elke i

bi = 0 of ci = 0 voor elke i.

Welnu, neem i ∈ {1, 2, . . . , k} willekeurig. Dan is wegens (∗) ai = 0 of bi = 0. Als ai = 0 dan is wegens (∗∗) en het feit dat ci ∈ N 0 ≤ ci ≤ ai = 0 zodat in dat geval ci = 0. We besluiten dat ai = 0 of bi = 0 voor elke i, zodat ggd(a, b) = 1.

XVI-36

(∗∗)


Voorlopig moeten we, om de positieve grootste gemene deler van a, b ∈ N0 te berekenen, eerst de standaardontbindingen van a en b bepalen. We hebben reeds opgemerkt dat daar geen efficiënt algoritme voor bekend is. Er is echter een simpele manier om ggd(a, b) te berekenen: het zogenaamde euclidisch algoritme. In dat proces wordt ggd(a, b) herleid tot ggd(m, n) voor zekere m, n die kleiner zijn dan a en b. Deze werkwijze steunt op de volgende 3 Stelling. Zij a, b ∈ Z niet beide nul. Dan geldt voor elke k ∈ Z: ggd(a, b) = ggd(a, ak + b) Bewijs. Neem k ∈ Z willekeurig.

We beweren dat ggd(a, b) | ggd(a, ak + b). Inderdaad, ggd(a, b) | a

en

ggd(a, b) | b

wegens definitie grootste gemene deler, deel (1)

ggd(a, b) | (a · k + b · 1)

wegens een basiseigenschap van deelbaarheid

ggd(a, b) | ggd(a, ak + b)

wegens definitie grootste gemene deler, deel (2) en het feit dat ggd(a, b) | a.

Daarnaast beweren we dat ook ggd(a, ak + b) | ggd(a, b). Inderdaad, ggd(a, ak + b) | a ⇒

en

ggd(a, ak + b) | (ak + b)

wegens definitie grootste gemene deler, deel (1)

ggd(a, ak + b) | (a · (−k) + (ak + b) · 1) {z } |

wegens een basiseigenschap van deelbaarheid

ggd(a, ak + b) | ggd(a, b)

wegens definitie grootste gemene deler, deel (2)

b

en het feit dat ggd(a, ak + b) | a.

Uit deze twee beweringen volgt nu: ggd(a, b) | ggd(a, ak + b)

en

ggd(a, ak + b) | ggd(a, b)

⇒ ggd(a, b) = ± ggd(a, ak + b) ⇒ ggd(a, b) = ggd(a, ak + b)

want ggd(a, b) > 0 en ggd(a, ak + b) > 0.

In het bijzonder is ggd(a, b) = ggd(a, b − a) en ggd(a, b) = ggd(a − b, b). Door dit herhaaldelijk toe te passen, kan het berekenen van ggd(a, b) herleid worden tot het berekenen van ggd(d, 0) voor een zeker geheel getal d. Dit proces wordt versneld door de deling met rest van het grootste getal door het kleinste getal uit te voeren. 3 Modelvoorbeeld 3. Bepaal ggd(195, 286) en schrijf als een lineaire combinatie van 195 en 286. Oplossing. We hebben: ggd(195, 286) = ggd(195, 286 − 195) = ggd(195, 91) = ggd(195 − 2 · 91, 91) = ggd(13, 91) = ggd(13, 91 − 7 · 13) = ggd(13, 0) = 13 waarbij de laatste gelijkheid volgt uit het feit dat 13 het grootste geheel getal is dat zowel 13 als 0 deelt. Door op onze stappen terug te keren, kunnen we 13 schrijven als een lineaire combinatie van 195 en 286: 13 = 195 − 2 · 91 = 195 − 2 · (286 − 195) = 3 · 195 + (−2) · 286.

XVI-37


4.2

Kleinste gemene veelvouden

Het begrip kleinste gemene veelvoud kent een grote analogie met dat van grootste gemene deler. De overeenkomstige eigenschappen kunnen dan ook op een gelijkaardige manier bewezen worden. Beschouw bijvoorbeeld de gehele getallen −12 en 8. Dan is −12Z ∩ 8Z = 24Z. Merk op dat ook −12Z ∩ 8Z = (−24)Z. Niet toevallig zijn 24 en −24 zogenaamde kleinste gemene veelvouden van −12 en 8, hetgeen betekent: (1) 24 en −24 zijn beide een veelvoud van −12 en 8, en (2) 24 en −24 zijn beide een deler van elk ander veelvoud c van −12 en 8. In het algemeen worden de kleinste gemene veelvouden van twee gehele getallen a en b als volgt gedefinieerd. 3 Definitie. Zij a, b ∈ Z0 . Een kleinste gemene veelvoud van a en b is een geheel getal v 6= 0 dat voldoet aan de volgende twee voorwaarden: (1) a | v en b | v,

(2) ∀c ∈ Z0 : a | c en b | c ⇒ v | c.

Voorbeeld. Een kleinste gemene veelvoud van −18 en 24 is −72. Een kleinste gemene veelvoud van 5 en 2 is 10. Het volgend resultaat wordt op een analoge manier bewezen als Eigenschap 1 voor grootste gemene delers uit de vorige paragraaf. Het bewijs laten we achterwege. 3 Eigenschap 1. Zij a, b ∈ Z0 en v ∈ Z0 . Dan zijn de volgende uitspraken equivalent: (i) v is een kleinste gemene veelvoud van a en b, (ii) vZ = aZ ∩ bZ. Een belangrijk gevolg van deze eigenschap is dat elk tweetal gehele getallen a, b beide verschillend van nul precies twee kleinste gemene veelvouden hebben, waarvan de ene tegengesteld is aan de andere. Het positieve kleinste gemene veelvoud van a en b zullen we met kgv(a, b) noteren. Merk dus op dat: kgv(a, b)Z = aZ ∩ bZ = (− kgv(a, b))Z. Kennen we van a, b ∈ Z0 de standaardontbinding, dan kunnen we net zoals bij ggd(a, b) nu ook kgv(a, b) op een eenvoudige manier vinden. Beschouw bijvoorbeeld a = 36 en b = 120. Dan is kgv(a, b) = kgv(36, 120) = 360. Nu kunnen we de standaardontbindingen van a, b en kgv(a, b) herschrijven als a = 36 = 22 · 32 · 50 b = 120 = 23 · 31 · 51 kgv(a, b) = 360 = 23 · 32 · 51 waarmee we inzien dat de volgende formule geldt: kgv(a, b) = 2max(2,3) · 3max(2,1) · 5max(0,1) . Opnieuw is deze formule eenvoudig te veralgemenen. Het bewijs, dat analoog is aan dat van Eigenschap 2 voor grootste gemene delers uit de vorige paragraaf, laten we als Oefening 23 voor de lezer. 3 Eigenschap 2. Zij a, b ∈ Z0 en stel a = e pa1 1 pa2 2 . . . pakk en b = e0 pb11 pb22 . . . pbkk waarbij e, e0 ∈ Z∗ , k ∈ N, ai , bi ∈ N en p1 < p2 < . . . < pk positieve priemgetallen zijn. Dan geldt: max(a1 ,b1 ) max(a2 ,b2 ) p2

kgv(a, b) = p1

max(ak ,bk )

. . . pk

Kennen we van twee gehele getallen hun standaardontbinding niet, dan moeten we op een andere manier hun positief kleinste gemeen veelvoud berekenen. Dat kan door een handig verband tussen ggd(a, b) en kgv(a, b) op te merken: is bijvoorbeeld a = 6 en b = 8, dan is: ggd(6, 8) = 2

en

kgv(6, 8) = 24

zodat

XVI-38

ggd(6, 8) · kgv(6, 8) = 48 = 6 · 8.


3 Stelling. Zij a, b ∈ N0 . Dan geldt:

ggd(a, b) · kgv(a, b) = ab

Bewijs. We herschrijven we de standaardontbinding van a en b als a = pa1 1 pa2 2 . . . pakk

b = pb11 pb22 . . . pbkk

en

waarbij k ∈ N, ai , bi ∈ N en p1 < p2 < . . . < pk positieve priemgetallen zijn. Dan is enerzijds: min(a1 ,b1 ) min(a2 ,b2 ) p2

ggd(a, b) · kgv(a, b) = p1

min(ak ,bk )

. . . pk

·

max(a1 ,b1 ) max(a2 ,b2 ) p2

p1

min(a1 ,b1 )+max(a1 ,b1 ) min(a2 ,b2 )+max(a2 ,b2 ) p2

= p1

max(ak ,bk )

. . . pk

min(ak ,bk )+max(ak ,bk )

. . . pk

terwijl anderzijds: a · b = pa1 1 pa2 2 . . . pakk

·

pb11 pb22 . . . pbkk

= p1a1 +b1 pa2 2 +b2 . . . pakk +bk .

Om de stelling aan te tonen, volstaat het om te bewijzen dat: min(ai , bi ) + max(ai , bi ) = ai + bi

voor elke i.

Welnu, neem i willekeurig. Dan is ai ≤ bi of ai > bi : . als ai ≤ bi dan is

min(ai , bi ) + max(ai , bi ) = ai + bi ; | {z } | {z } ai

bi

. als ai > bi dan is min(ai , bi ) + max(ai , bi ) = ai + bi . | {z } | {z } ai

bi

We besluiten dat min(ai , bi ) + max(ai , bi ) = ai + bi .

3 Modelvoorbeeld. Bepaal het kleinste positieve gemene veelvoud van 195 en 286. Oplossing. Met behulp van het euclidisch algoritme vinden we (zie Modelvoorbeeld 3 uit de vorige paragraaf): ggd(195, 286) = 13. Wegens de bovenstaande stelling is dan:

kgv(195, 286) =

=

195 · 286 ggd(195, 286) 195 · 286 13

= 195 · 22 = 4290.

XVI-39


4.3

Toepassingen

Priemgetallen zijn onlosmakelijk verbonden met de manier waarop gegegevens beveiligd worden. Heel wat belangrijke codeermechanismen steunen op de kennis van zeer grote priemgetallen. Tot op heden is er geen algoritme bekend dat priemgetallen op een efficiënte manier kan voortbrengen. Toch zijn wiskundigen al eeuwenlang op zoek geweest naar formules om priemgetallen te genereren. Zo vond Leonhard Euler [7] in 1772 dat n2 + n + 41 een priemgetal is voor n = 0, 1, 2, . . . , 39. En in 1837 bewees Johann Dirichlet [6] dat er in elke rekenkundige rij a, a + v, a + 2v, a + 3v, . . . met a, v ∈ Z0 relatief priem, oneindig veel priemgetallen voorkomen. Hierna beschrijven we priemgetallen van de vorm 2n − 1 en 2n + 1. In een laatste toepassing laten we zien hoe kleinste gemene veelvoud en grootste gemene deler een rol spelen bij de werking van tandwielen.

Toepassing 1 - Fermatgetallen Fermatgetallen zijn vernoemd naar Pierre de Fermat, die deze getallen voor het eerst bestudeerde. 3 Definitie Zij n ∈ N. Het n-de Fermatgetal is gelijk aan n

Fn = 22 + 1 3 Voorbeeld. De eerste zeven Fermatgetallen zijn gelijk aan: 0

F0 = 22 + 1 = 21 + 1 = 3 1

F1 = 22 + 1 = 22 + 1 = 5 2

F2 = 22 + 1 = 24 + 1 = 17 Pierre de Fermat (1601-1665)

3

F3 = 22 + 1 = 28 + 1 = 257 4

F4 = 22 + 1 = 216 + 1 = 65 537 5

F5 = 22 + 1 = 232 + 1 = 4 294 967 297 6

F6 = 22 + 1 = 264 + 1 = 18 446 744 073 709 551 617. In 1650 vermoedde Fermat dat elk Fermatgetal een priemgetal is, maar gaf toe dat hij dit niet kon aantonen. Zo zijn inderdaad de eerste vijf Fermatgetallen alle priem. Het vermoeden van Fermat werd weerlegd door Leonhard Euler 1732, die de standaardontbinding van het Fermatgetal F5 heeft berekend:3 F5 = 641 · 6 700 417. Intussen is aangetoond dat ook F6 , F7 , . . . , F32 geen priemgetallen zijn. Het is onbekend of er nog andere Fermatpriemgetallen zijn. Sommigen vermoeden dat het aantal Fermatpriemgetallen eindig is. In dat geval zou het aantal priemgetallen van de vorm 2n + 1 ook eindig zijn. Dit volgt uit de volgende eigenschap. 3 Eigenschap. Zij n ∈ N0 . Als 2n + 1 een priemgetal is, dan is n een macht van 2. Bewijs. Beschouw de veeltermidentiteiten (zie ook Oefening 10(b) uit Hoofdstuk 1): x3 + 1 = (x + 1)(x2 − x + 1) x5 + 1 = (x + 1)(x4 − x3 + x2 − x + 1) x7 + 1 = (x + 1)(x6 − x5 + x4 − x3 + x2 − x + 1) .. . xk + 1 = (x + 1)(xk−1 − xk−2 + xk−3 − · · · + x2 − x + 1) 3 Een

waarbij k > 1 oneven is.

uitspraak weerleggen betekent: met argumenten aantonen dat de uitspraak niet waar is.

XVI-40


We beweren dat n geen oneven deler heeft dat groter is dan 1. Inderdaad, mocht bijvoorbeeld 3 | n dan zou n = 3q voor een zekere q ∈ N0 en dan volgt uit de eerste identiteit: 2n + 1 = 23q + 1 = (2q )3 + 1 = (2q + 1)(22q − 2q + 1)

zodat (2q +1) | (2n +1). Omdat 1 < 2q +1 < 2n +1 volgt hieruit dat 2n +1 geen priemgetal is, een strijdigheid. Mocht bijvoorbeeld 5 | n dan zou n = 5q voor een zekere q ∈ N0 en dan volgt uit de tweede identiteit: 2n + 1 = 25q + 1 = (2q )5 + 1 = (2q + 1)(24q − 23q + 22q − 2q + 1)

zodat (2q +1) | (2n +1). Omdat 1 < 2q +1 < 2n +1 volgt hieruit dat 2n +1 geen priemgetal is, een strijdigheid. In het algemeen, mocht n een oneven deler k hebben dat groter is dan 1, dan zou n = kq voor een zekere q ∈ N0 en dan volgt uit de laatste identiteit: 2n + 1 = 2kq + 1 = (2q )k + 1 = (2q + 1)(2(k−1)q − 2(k−2)q + 2(k−3)q − · · · + 22q − 2q + 1)

zodat (2q + 1) | (2n + 1). Omdat 1 < 2q + 1 < 2n + 1 volgt hieruit dat 2n + 1 geen priemgetal is, een strijdigheid.

Hieruit volgt dat elke deler van n ofwel ±1 ofwel even is. Daaruit volgt dat n een macht van 2 is. Fermatgetallen kennen verschillende, onverwachte toepassingen. Zo heeft Carl Friedrich Gauss ingezien dat een regelmatige n-hoek met passer en liniaal kan worden geconstrueerd als en slechts als n een product is van een macht van 2 met een aantal verschillende Fermatpriemen. De bewering van Gauss werd in 1837 bewezen door Pierre-Laurent Wantzel [13]. In de volgende stelling vermelden we één van de belangrijkste eigenschappen van de rij van Fermatgetallen: een recursief voorschrift. Een gevolg hiervan is dat twee verschillende Fermatgetallen relatief priem zijn. Dit resultaat werd door Christian Goldbach in 1730 in een brief aan Leonhard Euler neergeschreven. 3 Stelling (Goldbach). Beschouw de rij van Fermatgetallen F0 , F1 , F2 , F3 , F4 , F5 , . . .. (a) Voor elke n ∈ N0 geldt: Fn = 2 + F0 · F1 · F2 · · · Fn−1 . (b) Zij Fm , Fn twee verschillende Fermatgetallen. Dan is ggd(Fm , Fn ) = 1. Bewijs. (a) We bewijzen de formule Fn = 2 + F0 · F1 · F2 · · · Fn−1 met inductie op n. (i) Inductiebasis. Voor n = 1 is enerzijds Fn = 5 en anderzijds is: 2 + F0 · F1 · F2 · · · Fn−1 = 2 + F0 = 2 + 3 = 5. Dus voor n = 1 is de formule aangetoond. (ii) Stel dat de formule waar is voor n = k (met k ≥ 1), dus dat Fk = 2 + F0 · F1 · F2 · · · Fk−1 . We moeten aantonen dat de formule waar is voor n = k + 1, dus we moeten aantonen dat Fk+1 = 2 + F0 · F1 · F2 · · · Fk . Welnu, 2 + F0 · F1 · F2 · · · Fk = 2 + F0 · F1 · F2 · · · Fk−1 · Fk = 2 + (Fk − 2) · Fk Ä k äÄ k ä = 2 + 22 + 1 − 2 22 + 1 Ä k äÄ k ä = 2 + 22 − 1 22 + 1 Ä k ä2 = 2 + 22 − 12 k

= 22

·2

+1

k+1

= 22

+1

= Fk+1 . Uit (i) en (ii) volgt nu dat de formule geldt voor alle n ≥ 1. XVI-41


(b) Zij Fm en Fn twee verschillende Fermatgetallen. Dan is m 6= n, dus m < n of m > n. Voor het vervolg van het bewijs zullen we aannemen dat m < n. Het afzonderlijk bewijs voor m > n is analoog. Welnu, uit (a) volgt : Fn = 2 + F0 · F1 . . . Fm . . . Fn−1 ⇒ 2 = Fn − F0 · F1 . . . Fm . . . Fn−1 . Nu is: ggd(Fn , Fm ) | Fn en ggd(Fn , Fm ) | Fm ⇒

ggd(Fn , Fm ) | (Fn − F0 · F1 . . . Fm . . . Fn−1 )

ggd(Fn , Fm ) | 2

ggd(Fn , Fm ) = 1 of ggd(Fn , Fm ) = 2.

Mocht ggd(Fn , Fm ) = 2 dan zou Ä n ä 2 | Fn ⇒ 2 | 22 + 1 zodat 2 een deler zou zijn van een oneven getal, een strijdigheid. We besluiten dat ggd(Fm , Fn ) = 1.

Omdat er oneindig veel Fermatgetallen zijn en elke twee Fermatgetallen onderling ondeelbaar zijn, volgt hieruit dat er oneindig veel priemgetallen zijn. Op die manier vinden we een ander bewijs voor de tweede stelling van Euclides, dat vreemd genoeg pas in 1891 werd gepubliceerd (en ontdekt?) door Adolf Hurwitz [8] en Hermann Scheffler [11, p.167], wellicht onafhankelijk van elkaar. 3 Stelling (tweede stelling van Euclides). Er zijn oneindig veel priemgetallen. Bewijs van Hurwitz en Scheffler. Omdat elk Fermatgetal groter is dan 1, bestaat er voor elk Fermatgetal Fn een priemgetal dat Fn deelt: kies een priemgetal p0 zodat p0 | F0 , kies een priemgetal p1 zodat p1 | F1 , kies een priemgetal p2 zodat p2 | F2 , .. . Welnu, beschouw de rij (pn ) = p0 , p1 , p2 , . . . Deze rij is een niet-eindigende opeenvolging van priemgetallen, gescheiden door komma’s. We beweren dat de termen in deze rij onderling verschillend zijn. Inderdaad, mocht pn = pm voor zekere n, m ∈ N met n 6= m, dan volgt:

Adolf Hurwitz (1859-1919)

pn | Fn en pn | Fm ⇒ pn | ggd(Fn , Fm ) ⇒ pn | 1

wegens de stelling van Goldbach,

strijdig met het feit dat pn een priemgetal is. Op die manier hebben we een (niet-eindigende) rij van onderling verschillende priemgetallen gemaakt. Hieruit volgt dat er oneindig veel priemgetallen zijn.

XVI-42


Toepassing 2 - Mersennegetallen Mersennegetallen zijn vernoemd naar Marin Mersenne, die een formule zocht om priemgetallen te genereren. 3 Definitie Zij n ∈ N0 . Het n-de Mersennegetal is gelijk aan Mn = 2n − 1 3 Voorbeeld. De eerste zeven Mersennegetallen zijn gelijk aan: M1 = 21 − 1 = 1

M5 = 25 − 1 = 31

M2 = 22 − 1 = 3

M6 = 26 − 1 = 63

M3 = 23 − 1 = 7

M7 = 27 − 1 = 127.

M4 = 24 − 1 = 15

Marin Mersenne (1588-1648)

Enkele voorbeelden doen ons vermoeden dat er een verband is tussen het al of niet priem zijn van een Mersennegetal Mn en het al of niet priem zijn van n. 3 Eigenschap. Zij n ∈ N0 . Als Mn = 2n − 1 een priemgetal is, dan is n een priemgetal. Bewijs. Beschouw de veeltermidentiteit (zie ook Oefening 10(a) uit Hoofdstuk 1): xk − 1 = (x − 1)(xk−1 + xk−2 + · · · + x + 1)

waarbij k ∈ N met k > 1.

We beweren dat n geen echte deler heeft dat groter is dan 1. Inderdaad, mocht n een echte deler k hebben dat groter is dan 1, dan zou n = kq voor een zekere q ∈ N met 1 < q < n en dan volgt uit de bovenstaande identiteit: 2n − 1 = 2kq − 1 = (2q )k − 1 = (2q − 1)(2(k−1)q + 2(k−2)q + · · · + 2q + 1) zodat (2q − 1) | (2n − 1). Omdat 1 < 2q − 1 < 2n − 1 volgt hieruit dat 2n − 1 geen priemgetal is, een strijdigheid. Het omgekeerde van de vorige eigenschap is niet waar: als n priem is dan hoeft Mn niet priem te zijn. Het kleinste tegenvoorbeeld is het Mersennegetal M11 = 211 − 1 = 2047 = 23 · 89.

In 1644 beweerde Mersenne dat Mn een priemgetal is voor n = 2, 3, 5, 7, 13, 17, 19, 31, 67, 127, 257 en een samengesteld getal is voor de andere waarden voor n met 1 ≤ n ≤ 257. De eerste fout in deze bewering werd pas in 1883 gevonden, toen Ivan Mikheevich Pervushin aantoonde dat M61 een priemgetal is. Vervolgens werden nog vier andere fouten in de bewering van Mersenne gevonden. In 1876 vond Éduard Lucas een manier om te testen of Mp een priemgetal is en gebruikte deze methode om aan te tonen dat M127 priem is. De Mersenne priemgetallen Mn met 1 ≤ n ≤ 257 worden gegeven door n = 2, 3, 5, 7, 13, 17, 19, 31, 61, 89, 107, 127. Het Mersennegetal M127 = 170 141 183 460 469 231 731 687 303 715 884 105 727 telt 39 cijfers en bleef het grootst gekende priemgetal tot Ferrier in 1951 met behulp van een mechanische rekenmachine vond dat F =

1 2148 + 1 = 20 988 936 657 440 586 486 151 264 256 610 222 593 863 921 17

een priemgetal is met 44 cijfers. Dit is meteen het grootste priemgetal dat zonder behulp van een elektronische rekenmachine of computer werd gevonden. Met de komst van de computers werden in het zelfde jaar al gauw grotere priemgetallen gevonden. Intussen werden 49 Mersenne priemgetallen gevonden.

de Arithmometer, een mechanische rekenmachine uit 1914

Op het moment dat deze tekst nagezien werd (15 augustus 2021) is het grootste priemgetal dat bekend is het Mersennegetal M82 589 933 . Het getal heeft 24 862 048 cijfers en werd gevonden op 7 december 2018.4 4 In 1996 werd de GIMPS (Great Internet Mersenne Prime Search) opgericht, een samenwerkingsproject van duizenden vrijwilligers die vrij beschikbare software gebruiken om nieuwe Mersenne priemgetallen op te sporen. Het project heeft tot op mei 2018 al zestien Mersenne priemgetallen ontdekt, waarvan het merendeel op het moment van hun ontdekking het grootst bekende priemgetal waren. Meewerken kan via http://www.mersenne.org/ . Een actuele lijst met de tot op heden bekende Mersenne priemgetallen is terug te vinden op http://www.mersenne.org/primes/ .

XVI-43


Toepassing 3 - Tandwielen Voor een draaiend tandwiel noteren we met r de straal (meter), z het aantal tanden (per omwenteling) en n het toerental (omwentelingen per minuut). Dan is de omtrek van het tandwiel 2πr, het aantal tanden per meter gelijk aan z/(2πr), en in een willekeurig vast punt op de rand (bijvoorbeeld de onderkant van het tandwiel) passeren er per minuut zn tanden. Beschouw nu twee tandwielen die in elkaar rollen (zie figuur). Willen de tandwielen niet slippen of blokkeren, dan moet het aantal tanden per meter op het eerste tandwiel gelijk zijn aan het aantal tanden per meter op het eerste tandwiel:5 z2 z1 = . 2πr1 2πr2 Verder zullen er, op de plaats waar de twee tandwielen in elkaar schuiven, per minuut evenveel tanden van het eerste tandwiel als van het tweede tandwiel passeren, zodat z1 · n1 = z2 · n2 . Hieruit besluiten we dat

z2 r2 n1 = = n2 z1 r1

Deze verhouding noemt men de overbrengingsverhouding (of transmissiecijfer of drijfwerkverhouding) van het eerste naar het tweede tandwiel, en wordt met i genoteerd. Is bijvoorbeeld i > 1, dan zal de as van het tweede tandwiel langzamer draaien dan de as van het eerste tandwiel, maar het tweede tandwiel levert wel een groter krachtmoment als gevolg van de grotere straal.

draaiende tandwielen met overbrengingsverhouding i = 40/16 = 2, 5

Het volgende modelvoorbeeld maakt een link tussen tandwielen en de begrippen grootste gemene deler en kleinste gemene veelvoud duidelijk. 3 Modelvoorbeeld. Twee tandwielen werken in op elkaar, het eerste heeft 64 tanden en doet 18 omwentelingen per minuut, het tweede tandwiel heeft 24 tanden. De straal van het tweede tandwiel is 10 cm. (a) Bepaal het toerental van het tweede tandwiel. (b) Bepaal de snelheid van een willekeurige tand op het eerste tandwiel en de snelheid van een willekeurige tand op het tweede tandwiel. (c) Na hoeveel omwentelingen van het kleinste tandwiel staan beide tandwielen voor de eerste keer weer in precies dezelfde stand? Maak gebruik van je grafische rekenmachine. Oplossing. (a) Noemen we het toerental van het tweede tandwiel n2 , dan volgt uit de overbrengingsverhouding i=

24 18 = 64 n2

dat n2 = 48. Het toerental van het tweede tandwiel bedraagt dus 48 omwentelingen per minuut. (b) Per omwenteling legt een tand op het tweede tandwiel een afstand van 2π ·0, 1 = 0, 2 π meter af. De snelheid van een willekeurige tand op het tweede tandwiel is dus gelijk aan v = 0, 2 π meter/omwenteling × 48 omwentelingen/minuut = 9, 6 π = 30, 15 . . . meter/minuut. Op de plaats waar de twee tandwielen in elkaar schuiven, zullen per minuut dan evenveel tanden van het eerste tandwiel als van het tweede tandwiel passeren. Daarom is de snelheid van een willekeurige tand op het eerste tandwiel ook gelijk aan v. (c) Noem k het aantal tanden waarover we het eerste tandwiel moeten verschuiven. Wil het eerste tandwiel in dezelfde stand staan, dan moet k ∈ 64Z. Over één tand in het eerste tandwiel verschuiven betekent ook dat we over één tand in het tweede tandwiel draaien. Wil ook het tweede tandwiel na k tanden in dezelfde stand staan, dan moet k ∈ 24Z. Zo vinden we dat k ∈ 64Z ∩ 24Z = kgv(64, 24)Z = 192Z. Het kleinste aantal tanden dat we het eerste (of het tweede) tandwiel moeten verschuiven is dus gelijk aan 192. Het tweede tandwiel telt minder tanden en is dus kleiner dan het eerste tandwiel. We besluiten dat het kleinste tandwiel 192/24 = 8 omwentelingen moet maken.

5 Strikt

genomen zijn de diameters 2r1 en 2r2 van de tandwielen niet de buitendiameters, maar wel de diameters van de raakcirkels.

XVI-44


Oefeningen 4 Grootste gemene delers en kleinste gemene veelvouden

Basis ?

Verdieping ? ??

??

4.1 Grootste gemene delers

1 2 3

4 5 6 7 8

7 8 9

4.2 Kleinste gemene veelvouden

17

18 19

20

4.3 Toepassingen

24 25 26

27 28

29

10 11 12

13 14

15 16

21 30

31 32

Uitbreiding ? ??

22

33

23 34

Oefeningen bij §4.1 B

Oefening 1. Zij a ∈ Z. Bewijs de volgende eigenschappen. (a) ggd(a, 0) = |a| voor a 6= 0

(c) ggd(a, a) = |a| voor a 6= 0

(b) ggd(a, 1) = 1

(d) ggd(a, a + 1) = 1

B

Oefening 2. Bewijs dat elke twee verschillende positieve priemgetallen relatief priem zijn.

B

Oefening 3 (Junior Wiskunde Olympiade 2013 eerste ronde). Wat is de grootste gemene deler van 2012 en 1220 ? (A) 2

B?

(B) 22

(C) 212

(D) 220

(E) 224

Oefening 4. Zij a ∈ Z en p een priemgetal. Bewijs dat ® ggd(a, p) =

p als p | a

1 als p6 | a.

B?

Oefening 5. Zij n ∈ N. Bewijs dat ggd(35n + 57, 45n + 76) ∈ {1, 19}.

B?

Oefening 6. Zij n ∈ N. Bewijs dat 5n + 3 en 7n + 4 onderling ondeelbaar zijn. Oefening 7. Bepaal telkens de positieve grootste gemene deler van a en b met behulp van het algoritme van Euclides. Schrijf daarna die positieve grootste gemene deler als een lineaire combinatie van a en b. B? (a) a = 1014 en b = 1404

B?? (c) a = 80 934 en b = 110 331

B? (b) a = 2268 en b = 3444

B?? (d) a = 147 231 en b = 839 160

Oefening 8. Zij a, b, c ∈ Z0 . Bewijs de volgende eigenschappen. B? (a) Als c | ab en ggd(a, c) = 1 dan is c | b. B? (b) Als a | c en b | c en ggd(a, b) = 1 dan is ab | c. B?? (c) ggd(ac, bc) = |c| ggd(a, b) B?? (d) Als d = ggd(a, b) dan is ggd(a/d, b/d) = 1. XVI-45


B??

Oefening 9. Zij a, b ∈ Z niet beide nul en d ∈ Z0 . Bewijs dat de volgende uitspraken equivalent zijn: (i) d is een grootste gemene deler van a en b, (ii) del(d) = del(a) ∩ del(b),

V

Oefening 10. Bewijs dat twee opeenvolgende kwadraten altijd relatief priem zijn.

V

Oefening 11. Schrijf 20151977 als een lineaire combinatie van 71 en 83.

V

Oefening 12 (Vlaamse Wiskunde Olympiade 2001 tweede ronde). De grootste gemene deler van 878 787 878 787 en 787 878 787 878 is (A) 3

(B) 9

(C) 27

(D) 10 101 010 101

(E) 30 303 030 303

V?

Oefening 13. Zij a, b ∈ Z0 en stel dat ggd(a, b) = 1. Vul aan en bewijs: ggd(a + b, a − b) = . . .

V?

Oefening 14. Zij n ∈ N0 en beschouw n + 1 verschillende natuurlijke getallen, alle groter dan nul en kleiner of gelijk aan 2n. Toon aan dat er minstens één tweetal van deze getallen is die relatief priem zijn.

U

Oefening 15 (grootste gemene deler van meer dan twee getallen). Zij a, b, c ∈ Z0 . Een grootste gemene deler van a, b en c is een geheel getal d 6= 0 dat voldoet aan de volgende twee voorwaarden: (1) d | a, d | b en d | c, (2) ∀e ∈ Z0 : e | a en e | b en e | c ⇒ d | e. Net zoals bij een grootste gemene deler van twee gehele getallen kan aangetoond worden dat d een grootste gemene deler van a, b en c is als en slechts als dZ = aZ + bZ + cZ. De positieve grootste gemene deler van a, b en c wordt met ggd(a, b, c) genoteerd. Analoog kan de grootste gemene deler van vier of meer gehele getallen worden gedefinieerd. (a) Toon aan dat ggd(a, b, c)Z = ggd(a, b)Z + cZ. (b) Bepaal ggd(13 320, 15 984, 19 980). Maak gebruik van je grafische rekenmachine.

U

Oefening 16 (Diophantische vergelijkingen). Een Diophantische vergelijking is een vergelijking waarbij enkel de oplossingen worden onderzocht die gehele getallen zijn. De meest eenvoudige Diophantische vergelijkingen zijn de lineaire veeltermvergelijkingen in één of meerdere onbekenden. Zij a, b, c ∈ Z0 . Vul telkens de uitspraak aan en bewijs. (a) De Diophantische vergelijking ax = c heeft oplossingen als en slechts als . . . | c. (b) De Diophantische vergelijking ax + by = c heeft oplossingen als en slechts als . . . | c.

Oefeningen bij §4.2 B

Oefening 17. Waar of vals? Beoordeel de volgende uitspraak. Indien vals, geef een tegenvoorbeeld. Indien waar, bewijs. Zij a, b ∈ Z0 en v ∈ Z0 . Dan zijn de volgende uitspraken equivalent: (i) v is een kleinste gemene veelvoud van a en b, (ii) del(v) = del(a) ∪ del(b),

B?

Oefening 18. Bepaal het positieve kleinste gemene veelvoud van 2204 en 4840 met behulp van het algoritme van Euclides.

B?

Oefening 19. Bepaal alle natuurlijke getallen waarvoor het positieve kleinste gemene veelvoud van dat getal en 30 gelijk is aan 450.

XVI-46


B??

Oefening 20. Beschouw de gehele getallen a = 30 031 en b = 16 579. (a) Bepaal ggd(a, b). (b) Schrijf ggd(a, b) als een lineaire combinatie van a en b. (c) Bepaal kgv(a, b).

V??

Oefening 21 (Nederlandse Wiskunde Olympiade 1998 finale). Van twee positieve gehele getallen m en n is het kleinste gemeenschappelijke veelvoud gelijk aan 133 866 = 2·3·3·3·37·67. Het verschil m − n is gelijk aan 189. Bereken m en n.

U

Oefening 22 (kleinste gemene veelvoud van meer dan twee getallen). Zij a, b, c ∈ Z0 . Een kleinste gemene veelvoud van a, b en c is een geheel getal v 6= 0 dat voldoet aan de volgende twee voorwaarden: (1) a | v, b | v en c | v, (2) ∀d ∈ Z0 : a | d en b | d en c | d ⇒ v | d. Net zoals bij een kleinste gemene veelvoud van twee gehele getallen kan aangetoond worden dat v een kleinste gemene veelvoud van a, b en c is als en slechts als vZ = aZ ∩ bZ ∩ cZ. Het positieve kleinste gemene veelvoud van a, b en c wordt met kgv(a, b, c) genoteerd. Analoog kan het kleinste gemene veelvoud van vier of meer gehele getallen worden gedefinieerd. (a) Toon aan dat kgv(a, b, c)Z = kgv(a, b)Z ∩ cZ. (b) Bepaal kgv(9009, 15 039, 18 942). Maak gebruik van je grafische rekenmachine.

U?

Oefening 23 (eigenschap 2). Zij a, b ∈ Z0 en stel a = e pa1 1 pa2 2 . . . pakk en b = e0 pb11 pb22 . . . pbkk waarbij e, e0 ∈ Z∗ , k ∈ N, ai , bi ∈ N en p1 < p2 < . . . < pk positieve priemgetallen zijn. Bewijs dat max(a1 ,b1 ) max(a2 ,b2 ) p2

kgv(a, b) = p1

max(ak ,bk )

. . . pk

.

Oefeningen bij §4.3 B

Oefening 24. Zij a, n ∈ N, a ≥ 2 en stel dat p = an + 1 een priemgetal is. Bewijs dat a even is.

B

Oefening 25 (Vlaamse Wiskunde Olympiade 2011 eerste ronde). Welk van de volgende tandwielen, die elk rond hun eigen (vaste) as draaien, maakt de meeste omwentelingen per minuut?

(A) I

B

(B) II

(C) III

(D) IV

(E) V

Oefening 26. De omtrek van de voorwielen van een rijtuig is 2, 75 m, die van de achterwielen 3, 30 m. Welke is de kleinste strikt positieve afstand die het rijtuig moet afleggen als de voor- en achterwielen op hetzelfde ogenblik een geheel aantal toeren moeten gedaan hebben?

XVI-47


B?

Oefening 27. Gegeven is onderstaande ketting van tien tandwielen. Van elk tandwiel is het aantal tanden aangeduid. Bepaal de overbrengingsverhouding van het eerste tandwiel (links) naar het laatste tandwiel (rechts).

B?

Oefening 28. Twee tandwielen werken in op elkaar. Het eerste tandwiel heeft een straal van 21 cm en maakt 635 omwentelingen per minuut. De straal van het tweede tandwiel is gelijk aan 15 cm. (a) Bepaal de hoeksnelheid (radialen per minuut) ω1 en ω2 van beide tandwielen. (b) Na hoeveel minuten zullen beide tandwielen voor de eerste keer een geheel aantal omwentelingen gemaakt hebben? Geef de exacte waarde. Maak gebruik van je grafische rekenmachine.

B??

Oefening 29. De zijden van twee vierkante tuinen hebben elk een lengte die een geheel veelvoud is van 3 m, 3, 5 m en 4 m. Het verschil tussen de twee oppervlakten bedraagt 3, 528 ha.6 Bereken de lengte van de zijde van elke tuin.

V

Oefening 30. Sinds 1997 is er in Groot-Brittanië een nieuw muntstuk van twee pond in omloop. De ene zijde beeldt Queen Elizabeth II uit, de ommezijde werd ontworpen door Bruce Rushin en symboliseert de technologische vooruitgang door de eeuwen heen. Er is een gesloten ketting van 19 werkende tandwielen te zien (zie figuur). Welke blunder zag men blijkbaar over het hoofd?7

V?

Oefening 31. Zij a, n ∈ N, beide groter dan 1, en stel dat an − 1 een priemgetal is. Bewijs dat an − 1 een Mersennegetal is.

6 Een

hectare is een eenheid van oppervlakte van 100 m × 100 m = 10 000 m2 en wordt afgekort als ha. In een hectare gaan 100 aren. klachtenbrief aan de Royal Mint of Britain werd als volgt beantwoord: “We would, however, wish to emphasise that this is a coin design symbolic of the development of technology and its success lies in visually representing a complex idea in an interesting and succinct fashion.” Naar schatting zijn momenteel zo’n 394 miljoen muntstukken van twee pond in omloop. 7 Een

XVI-48


V?

Oefening 32. Vier tandwielen, met respectievelijk vijf, zeven, elf en dertien tanden, grijpen op elkaar in zoals in de figuur hiernaast. De vaste pijlen wijzen telkens naar het getal op de bovenste tand, in de beginstand dus 2 0 0 6. Je kan aan elk van de tandwielen draaien, om zo een ander getal in de bovenste positie te zetten. Uiteraard zullen de andere tandwielen dan meedraaien en ook een ander getal aanwijzen. Is het mogelijk de huidige aanduiding 2 0 0 6 door draaiing te veranderen in 2 0 0 7? En zo ja: hoe vaak en in welke richting moet het bovenste tandwiel dan worden rondgedraaid?

V??

Oefening 33. Als men de treden van een trap twee aan twee telt, blijft er 1 over. Telt men ze drie aan drie, dan blijven er 2 over. Telt men ze vijf aan vijf, dan blijven er 4 over. En telt men ze zeven aan zeven, dan blijven er 6 over. Bepaal alle mogelijkheden voor het aantal treden als je weet dat het aantal kleiner dan 1000 is. poster Vlaamse Wiskunde Olympiade, 2006

U??

Oefening 34 (slijtage bij tandwielen). Is er bij samenwerkende tandwielen één tand oneffen of beschadigd, dan zal het andere tandwiel na verloop van tijd slijtage vertonen. Komt de beschadigde tand voortdurend met dezelfde tand of paar tanden in aanraking, dan zal die slijtage zich op één of enkele punten fixeren. Uiteraard wil men dat de tandwielen zo lang mogelijk meegaan, dus moet men ervoor zorgen dat de slijtage gelijkmatig optreedt. De optimale manier is om ervoor te zorgen dat er zoveel mogelijk tanden in aanraking met de beschadigde tand komen. In onderstaande figuur werken twee tandwielen samen. Het kleinste tandwiel heeft z1 = 20 tanden en het grootste tandwiel heeft z2 = 30 tanden. De tanden zijn geletterd en genummerd van A tot en met T, resp. van 1 tot en met 30. Tand A van het kleinste tandwiel drukt op een bepaald moment tegen tand 1 van het grote tandwiel. (a) Met welke tanden komt tand A nog meer in aanraking als het een zeer groot aantal keer ronddraait? (b) Zij r een geheel getal met 0 ≤ r < 30. Toon aan dat tand A in aanraking met tand r komt als en slechts als r − 1 een veelvoud van 10 is. (c) Stel dat het grote tandwiel z2 = 25, 29 of 21 tanden heeft. Met welke tanden komt tand A dan in aanraking? (d) Toon aan dat elke tand van het ene tandwiel met elke tand van het andere tandwiel in aanraking komt als en slechts als het aantal tanden van het ene tandwiel relatief priem is met het aantal tanden van het andere tandwiel.

XVI-49


Antwoorden op geselecteerde oefeningen Hoofdstuk 1 (1) (a) {3, 6}

(b) {1, 2, 3, 4, 5, 6} (c) {1, 5}

(d) {2, 4} (e) ∅

(f) {1, 5}

(2) (a) vals (b) waar (c) waar (d) vals (e) waar (f) vals (g) waar (h) vals (3) (a) {−5, 5}

(b) {1, 3, 5, 7, 9, . . .} (c) ∅

(4) (a) Q = ∅

(b) P = ∅ en Q = ∅ (c) P = Q

(d) P = Z en Q = ∅ (5) (D) (6) (a) V = {−4, −3, −2, −1, 0, 1, 2, 3}

(b) De verzameling V bevat een eindig aantal elementen en #V = 8.

(7) (a) vals (b) vals (c) vals (d) waar (e) vals (f) vals (g) vals

XVI-50


Hoofdstuk 2 (1) n ∈ {1, −1, 2, −2, 3, −3, 4, −4, 6, −6, 12, −12} (2) (a) vals (b) vals (c) vals (d) vals (e) vals (4) (C) (5) (a) a = 10 of a = −10

(b) Zij a, b ∈ Z. Dan geldt: del(a) = del(b) ⇔ a = b of a = −b.

(10) (a) Het quotiënt is 8 en de rest is 4. (b) Het quotiënt is −9 en de rest is 3. (c) Het quotiënt is −7 en de rest is 1.

(d) Het quotiënt is −3 en de rest is 3. (e) Het quotiënt is −8 en de rest is 0. (f) Het quotiënt is 0 en de rest is 15.

(11) {90, 91, 92, 93, 94} (12) De deler is −25 en het deeltal is −600. (13) 1098 (14) Het quotiënt is 12 en de rest is 8. Het gezochte geheel getal is 14. (15) (a) −102 529 (b) −104 664

(17) De deler is gelijk aan 8. (19) Het natuurlijk getal moet strikt tussen 40 en 99 liggen. (20) (E) (21) Ofwel is het deeltal gelijk aan 247 en de deler gelijk aan 19, ofwel is het deeltal gelijk aan 248 en de deler gelijk aan 18. (23) (a) waar (b) vals (26) (C) (30) De rest is gelijk aan 18.

Hoofdstuk 3 (1) 4, 6, 8, 9, 10, 12, 14 (2) n ∈ {1, −1, p, −p} Å ã Å ã Å ã Å ã (3) a = 40, b = 39 of a = −40, b = −39 of a = 40, b = −39 of a = 40, b = −39 (4) (a) n = 1 (b) n = 2 (c) n = 1 of n = 3 (d) Voor geen enkel natuurlijk getal n is p een priemgetal. (6) (A) (11) −2310 XVI-51


(12) n = 7350 (13) (a) −899 = (−1) · 29 · 31 (b) 1485 = 33 · 5 · 11

(c) −5820 = (−1) · 22 · 3 · 5 · 97

(d) 9075 = 3 · 52 · 112 (14) (D) (15) 0 (21) (b) 144

Hoofdstuk 4 (3) (E) (7) (a) ggd(1014, 1404) = 78 = 7 · 1014 − 5 · 1404

(b) ggd(2268, 3444) = 84 = 2 · 3444 − 3 · 2268

(c) ggd(80 934, 110 331) = 123 = −223 · 110 331 + 304 · 80 934

(d) ggd 147 231, 839 160) = 189 = −263 · 103 005 + 184 · 147 231 (11) 20151977 = 71 −7 · 20151977 + 83 6 · 20151977 (12) (E) (13) ggd(a + b, a − b) = 1 of ggd(a + b, a − b) = 2 (14) (b) 1332 (16) (a) a | c

(b) ggd(a, b) | c

(18) 2 666 840 (19) 225 en 450 (20) (a) 59 (b) 59 = 53 · 30 031 + (−96) · 16 579 (c) 30 171 600

(21) m = 1998 en n = 1809 (22) (b) 1 234 431 198 (25) (C) (26) 1650 m (27) 3 (28) (a) ω1 = 1270π radialen/minuut en ω2 = 1778π radialen/minuut (b) 1/127 minuten (29) De zijde van de grootste tuin meet 252 m en de zijde van de kleinste tuin meet 168 m. (32) Het bovenste tandwiel moet 1925 tanden in tegenwijzerzin gedraaid worden, oftewel 385 omwentelingen in tegenwijzerzin. (33) {209, 419, 629, 839} (34) (a) Tand A komt in aanraking met tanden 1, 21 en 11. (c) Tand A komt respectievelijk in aanraking met tanden 1, 6, 11, 16, 21 en 1 en 1, 2, 3, 4, . . . , 21.

XVI-52


Referentielijst [1] M. Aigner, G.M. Ziegler, Proofs from the book, Springer, 1998. [2] C.G. Bachet de Méziriac, Probl‘emes plaisants et délectables qui se font par les nombres, partie recueillis de divers autheurs, et inventez de nouveau, avec leur démonstration, seconde édition, revue, corrigée et augmentée de plusieurs propositions et de plusieurs problm̀es, Pierre Rigaud & Associés, Lyon, 1624. [3] É. Bézout, Théorie générale des équations algébriques, Imprimerie Ph.-D. Pierres, Paris, 1779. [4] K. De Naeghel, Vijf minuten wiskunde: een verzameling van kortverhalen, print-on-demand online publishing Issuu.com, 2018. Beschikbaar op https://issuu.com/koendenaeghel/docs/vijfminutenwiskundedeel . [5] K. De Naeghel, L. Van den Broeck, SOHO Wiskunde Plantyn Lineaire Algebra I, Plantyn, 2014. [6] G.L. Dirichlet, Beweis des Satzes, dass jede unbegrenzte arithmetische Progression, deren erstes Glied und Differenz ganze Zahlen ohne gemeinschaftlichen Factor sind, unendlich viele Primzahlen enthält, Abhandlungen der Königlichen Preußischen Akademie der Wissenschaften zu Berlin, Vol. 48, p. 45-71, 1837. [7] L. Euler, Nouveaux Mémoires de l’Acadḿie royale des Sciences, Berlin, p. 36, 1772. [8] R. Haas, Goldbach, Hurwitz, and the Infinitude of Primes: Weaving a Proof across the Centuries, Math Intelligencer, Vol. 36, p. 1, 2014. [9] S. Northshield, A One-Line Proof of the Infinitude of Primes, The American Mathematical Monthly, Vol. 122(5), p. 466-466, 2015. [10] R. Rivest, A. Shamir, L. Adleman, A Method for Obtaining Digital Signatures and Public-Key Cryptosystems, Communications of the ACM. 21(2), p. 120126, 1978. Online versie beschikbaar op http://people.csail.mit.edu/rivest/Rsapaper.pdf . [11] H. Scheffler, Beiträge zur Zahlentheorie, insbesondere zur Kreis- und Kugeltheilung, mit einem Nachtrage zur Theorie der Gleichungen, Verlag von Friedrich Foerster, Leibzig, 1891. [12] A. Turing, On computable numbers, with an application to the Entscheidungsproblem, Proceedings of the London Mathematical Society, Series 2, Volume 42, p. 230-265, 1937. Online versie beschikbaar op http://www.turingarchive.org/browse.php/B/12 . [13] P-L. Wantzel, Recherches sur les moyens de reconnaı̂tre si un Problème de Géométrie peut se résoudre avec la règle et le compas, Journal de Mathématiques Pures et Appliquées, Vol. 1(2), p. 366-372, 1837.

XVI-53


Issuu converts static files into: digital portfolios, online yearbooks, online catalogs, digital photo albums and more. Sign up and create your flipbook.